Sunteți pe pagina 1din 51

Universidad La Salle.

Facultad Mexicana de Medicina Curso de Extensin Universitaria para la Preparacin del Examen Nacional para Aspirantes a Residencias Mdicas. Examen Final. 01 Agosto del 2009 Modalidad Presencial.

1. Fem. 39 aos acude al servicio de urgencias quejndose de calambres en las piernas y parestesias en los dedos de las manos. Un ao antes se le someti a una operacin del cuello, pero no est segura qu fue lo que se le hizo. El signo de Chevostek es positivo: la percusin sobre el nervio facial por delante de la oreja desencadena una contraccin espasmdica del labio superior. El siguiente trastorno sugiere el signo de chevostek? (A) (B) (C) (D) Hipercalciemia Hipocalciemia Hiperpotasemia Acidosis

Allen R. M. MMS Medicina Interna. 5. Edicin. National Medical Series. Mc. Graw Hill. 2006. (captulo 9 III B 3 a (1) (b)). El signo del sndrome Chevostek sugiere hipocalciemia. La disminucin del calcio ionizado provoca mayor irritabilidad neuromuscular. Adems del signo de Chevostek, los pacientes pueden tener entumecimiento y hormigueo de las extremidades, fasciculaciones y calambres musculares, y fatiga y debilidad musculares. La hipocalciemia grave, si no se trata, puede causar laringoespasmo y convulsiones. 2. Fem 50 aos, originaria de Granada, Espaa y radicada en la Ciudad de Mxico refiere rubor, tumefaccin y rigidez de las articulaciones interfalngicas dstales desde hace tres meses, El diagnsticos de esta paciente es? (A) Osteoartritis erosiva (B) Artritis reumatoide (C) Lupus eritematoso diseminado (D) Espondilitis anquilosante

Allen R. M. MMS Medicina Interna. 5. Edicin. National Medical Series. Mc. Graw Hill. 2006. (captulo 10 V E 1). Tpicamente, la osteoartritis erosiva afecta las articulaciones interfalngicas dstales en mujeres de edad madura. Es improbable que esos sntomas articulares dstales prominentes sucedan en pacientes con artritis reumatoide o con lupus eritematoso diseminado sin molestias articulares ms generalizadas. No hay pruebas que indiquen espondilitis anquilosante o esclerodermia.

3. Masculino 50 aos, con insuficiencia renal crnica debida a hipotensin prolongada grave acude a urgencias porque presenta dolor retroesternal. Se le indic hemodilisis dos veces por semana en los ltimos dos aos y en fechas recientes ha experimentado episodios de hipotensin al inicio del tratamiento. El dolor se localiza sobre el msculo trapecio. Se reduce un poco al adoptar la posicin de pie y se exacerba con la respiracin profunda. Causa probable del dolor retroesternal de este paciente? (A) Pericarditis (B) Artropata coronaria (C) Espasmo esofgicos difusos (D) Embolias pulmonares

Allen R. M. MMS Medicina Interna. 5. Edicin. National Medical Series. Mc. Graw Hill. 2006. (captulo 6, parte I: III E 2). El dolor torcico que sufri este paciente es caracterstico de pericarditis e inflamacin del pericardio, complicaciones comunes en personas con insuficiencia renal crnica en hemodilisis. Estos enfermos tambin pueden tener inflamacin de varios recubrimientos serosos, como peritoneo y pleura; no se conoce el mecanismo de esta complicacin. Aunque es comn la arteriopata coronaria en pacientes en dilisis, las caractersticas de dolor en este individuo sugieren que no es el diagnstico. La enfermedad esofgica tambin es comn en sujetos en dilisis y debe descartarse especficamente como posible causa. La relacin aparente con la dilisis, as como la frecuencia de los sntomas, van contra el diagnstico de embolia pulmonar como causa del dolor torcico. Adems, el dolor musculoesqueltico debido a diversos trastornos se observa en pacientes en dilisis y puede deberse a anomalas en el metabolismo del calcio y el fsforo, que produce depsitos de calcio en diversos componentes del sistema musculoesqueltico. 4. Masculino 30 aos adicto a las drogas intravenosas acude a sala de urgencias por presentar debilidad del hemicuerpo derecho y cefalea en un periodo de dos das. La exploracin revela un individuo mal nutrido y afebril con hemiparesia derecha leve. El Diagnstico ms probable es: ? (A) Endocarditis bacteriana (B) Meningitis por virus de la inmunodeficiencia humana (VIH) (C) Absceso cerebral (D) Meningitis criptoccica.

Allen R. M. MMS Medicina Interna. 5. Edicin. National Medical Series. Mc. Graw Hill. 2006. (Captulo 11 XVI B 1-2). Los adictos a drogas intravenosas estn propensos a sufrir bacteriemia, que a su vez puede producir absceso cerebral y disfuncin neurolgica progresiva. Los pacientes con aqul, por lo comn se encuentran afebriles a no ser que haya endocarditis acompaante u otro origen endovascular de infeccin. Los adictos a drogas intravenosas tienden a presentar endocarditis bacteriana y pueden cursar con dficit neurolgicos en forma apopltica debido a embolia sptica del cerebro. Sin embargo, por lo regular tienen fiebre. La

meningitis por VIH produce cefalea y datos de irritacin menngea, pero no se presenta dficit neurolgico focal. La meningitis criptoccica se manifiesta con conducta alterada y cefalea y los pacientes estn afebriles. Sin embargo, es raro que haya datos de seudoapopleja. Por ltimo, el uso de drogas intravenosas puede provocar embolia de cuerpo extrao pero con problemas neurolgicos apoplticos. Un mbolo puede llegar al cerebro por un cortocircuito cardaco de derecha a izquierda o bien por una malformacin arteriovenosa pulmonar si la inyeccin es venosa. El mbolo puede entrar a la circulacin cerebral de manera directa en caso de inyeccin intracarotdea.

5. Masculino 45 aos con cirrosis heptica tuvo dolor abdominal generalizado durante 24 h sin nuseas ni vmitos. Su temperatura es de 38.3C y ha tenido distensin abdominal con onda de lquido claro. Hay hipersensibilidad difusa en la palpacin abdominal. En la paracentesis se obtuvo lquido transparente con 816 leucocitos/mm3 (85% polimorfonucleares, 15% linfocitos). La tincin de Gram no muestra bacterias. La entidad nosolgica en este paciente es? (A) (B) (C) (D) Enfermedad ulcerosa pptica Peritonitis primaria Pancreatitis Colecistitis

Allen R. M. MMS Medicina Interna. 5. Edicin. National Medical Series. Mc. Graw Hill. 2006. (captulo 8 V E 1 a). El diagnstico ms probable es peritonitis primaria. Aunque es difcil diferenciar la peritonitis primaria (espontnea) por rotura de vscera hueca y contaminacin peritoneal, la presencia de fiebre y la leucocitosis la ascitis sugiere alguna clase de infeccin peritoneal. La pancreatitis se caracteriza por dolor localizado intenso (mesoepigstrico), que se irradia a la espalda. En general, las nuseas y vmitos no se relacionan con pancreatitis aguda. El dolor abdominal en caso de colecistitis se sita en el cuadrante superior derecho y suele haber nuseas y vmitos. El absceso heptico tiende a ser un trastorno subagudo sin datos peritoneales prominentes. En caso de ascitis crnica infectada, las enfermedades ulcerosas ppticas son causa poco probable. 6. En los estudios epidemiolgicos existe un modelo retrospectivo, observacional, comparativo, que parte del efecto a la causa, que es til cuando se investigan enfermedades de baja incidencia, que no expone a riesgo a ninguno de los sujetos estudiados y que ofrece un ndice conocido como riesgo relativo (Odds Ratio). Ese diseo es: (A) (B) (C) (D) La serie de casos El reporte epidemiolgico Ensayo clnico controlado Casos y controles

Crdova VH, Jimnez J, Jimnez MC. Manual de diseo metodolgico en investigacin clnica. ULSA UAPY 2001. Pag 30 31. Los estudios de casos y controles son retrospectivos, de observacin y comparativos. A diferencia de los estudios de Cohorte, parten en esencia desde

el efecto y hacia atrs buscan identificar causas o factores de riesgo o exposicin. Al grupo de personas que tienen el fenmeno o enfermedad se le denomina casos y se compara con otro grupo de individuos que no tienen el fenmeno o enfermedad y se les denomina controles Este tipo de diseo es particularmente til cuando queremos estudiar a pacientes o poblaciones con alguna enfermedad que se present mucho tiempo despus de haber ocurrido la exposicin o cuando queremos investigar los factores de riesgo de enfermedades poco frecuentes o con causas mltiples. En estos estudios se calcula el riesgo relativo (Odds Ratio) y se interpreta en razn al valor de 1, esto es, menos de uno es poco probable que ocurra el riesgo relativo y ms de uno es probable que s ocurra.

7. Masculino 65 aos lee en el peridico que el antgeno prosttico especfico es una buena prueba de investigacin para el cncer y pide a su internista que se la haga. La prueba revela un aumento de antgeno prosttico especfico de 10.4 ng/ml. El tacto rectal revela una prstata de tamao normal, pero en la ecografa se encuentra un rea hipoecoica pequea que mide 5x7 mm en el lbulo derecho. El examen clnico mas adecuado en este paciente es? (A) Realizar gammagrama seo (B) Repetir el anlisis de antgeno prosttico especfico en tres meses para verificar si hay aumento ulterior (C) Practicar biopsia transrectal del rea anormal encontrada en la ecografa (D) Comenzar el tratamiento con leuprolida de depsito

Allen R. M. MMS Medicina Interna. 5. Edicin. National Medical Series. Mc. Graw Hill. 2006. (Captulo 4 X C 4 a (3), b (1)). El antgeno prosttico especfico es una prueba que puede usarse para deteccin de cncer de prstata. Sin embargo, los valores de ste tambin pueden estar un poco altos en la hipertrofia prosttica benigna. Una ecografa transrectal puede identificar lesiones pequeas no palpables en la exploracin rectal. Si los pacientes tienen PSA alto y se confirm en la ecografa un rea anormal, se puede practicar biopsia por va transrectal bajo gua ecogrfica. En general, a los varones con cncer de prstata se les estudia en busca de metstasis con gammagrama seo; radiografa de trax; tomografa computarizada de retroperitoneo y pelvis; o resonancia magntica de retroperitoneo y pelvis con nfasis en la prstata, acompaados por estudios de laboratorio. El carcinoma prosttico metastsico puede tratarse con leuprolide, un agonista de la hormona liberadora de hormona luteinizante que suprime la produccin testicular de testosterona. El tratamiento con leuprolide equivale a orquiectoma o a teraputica con estrgenos en cncer prosttico o metastsico. 8. Masculino 60 aos de edad, acude a consulta externa asegura ser hipertenso de ms de cinco aos de evolucin, sin agudizaciones adems de ser portador de una fibrilacin auricular, motivo por los cuales recibe propranolol en dosis de 100 mgs cada maana. El hallazgo de laboratorio en este paciente es? (A) Ausencia de onda P, complejo QRS normal e intervalos R-R diferentes.

(B) Ausencia de onda P, complejo QRS ensanchado, diferencia constante de los intervalos R-R. (C) Presencia de onda P, complejo QRS ensanchado y eje rotado a la derecha. (D) Ritmo nodal con disociacin A-V

Guadalajara J. Cardiologa. Sexta Edicin 151 152 La fibrilacin auricular es la arritmia cardiaca ms frecuente. La despolarizacin catica y desordenada de las aurculas por mltiples ondas en simultnea, trae como consecuencia que se pierda la funcin mecnica de la contraccin auricular. Estos dipolos de activacin mltiple y desordenados alcanzan al nodo A-V y penetran en l, algunos pasan hacia el Haz de His mientras que otros no lo alcanzan, debido a la penetracin parcial del nodo por conduccin decreciente completa, esto se conoce como, conduccin oculta; as, la rpida penetracin de los estmulos auriculares favorece la aparicin de conduccin oculta, la cual afecta en forma impredecible el periodo refractario del nodo. El trazo electrocardiogrfico tpico es la ausencia de onda P, complejo QRS normal e intervalos R-R diferentes.

9. Aunque la mayor parte de los casos de hepatitis vrica aguda cede espontneamente, pueden sobrevenir complicaciones. Cul de las siguientes aseveraciones describe mejor las complicaciones de la infeccin por virus de hepatitis B (HBV)? (A) El estado de portador crnico coexiste con mayor riesgo de hepatoma (B) Por lo comn la hepatitis persistente crnica provoca deterioro progresivo del funcionamiento heptico y debe tratarse enrgicamente (C) La hepatitis activa crnica puede diagnosticarse en dos a cuatro semanas de la infeccin aguda con HBV (D) La hepatitis activa crnica se caracteriza en la biopsia heptica por infiltrado linfoctico periportal sin fibrosis o extensin extraportal

Allen R. M. MMS Medicina Interna. 5. Edicin. National Medical Series. Mc. Graw Hill. 2006. (captulo 5IX A1 f (1 )-(3)). El estado de portador crnico del virus de hepatitis B (HBV) coexiste con mayor riesgo de hepatoma y se encuentra en 0.2% de la poblacin estadounidense. No es posible diagnosticar hepatitis crnica activa hasta por lo menos seis meses despus de la infeccin aguda con HBV. En la hepatitis activa crnica, la inflamacin y la fibrosis rebasan el rea portal y por tanto, se correlacionan con deterioro profuso de la funcin heptica, que puede resultar en cirrosis o insuficiencia heptica. Se presenta hepatitis fulminante en alrededor de 1 a 2% de los casos de HBV, hepatitis C (HCV) y hepatitis no A, no B y no C. Esta complicacin rara por lo regular se relaciona con baja de las concentraciones de transaminasa conforme es destruido el tejido heptico y disminuye el tamao del hgado.

10. Fem. 80 aos antecedentes de insuficiencia cardiaca congestiva presenta angina de


pecho. Sus medicamentos se ajustan con: furosemida, digoxina, nitroglicerina y potasio complementario. Poco despus presenta cefaleas pulstiles intermitentes. Medida Inmediata a tomar en esta paciente?:

(A) Realizar biopsia de arteria temporal (B) Comenzar la administracin de propranolol (C) Comenzar la administracin de ergotamina sublingual (D) Suspender la nitroglicerina Allen R. M. MMS Medicina Interna. 5. Edicin. National Medical Series. Mc. Graw Hill. 2006. (captulo 11IV B 1 a (2), d (1) (a), 4 a-b). La nitroglicerina puede causar cefaleas "vasculares" terebrantes; por tanto, la ms sencilla opcin de tratamiento es suspender el preparado de nitroglicerina y utilizar, si es posible, un medicamento cardaco alternativo. Siempre debe considerarse la arteritis temporal como posible causa de cefalea en pacientes mayores de 50 aos. Favorecen este diagnstico el aumento de la velocidad de eritrosedimentacin; la claudicacin mandibular; las artralgias y las mialgias; y una arteria temporal hipersensible e indurada. El propranolol es un antimigraoso eficaz. Sin embargo, antes de prescribir estos frmacos deben descartarse los posibles factores que la precipitan. La ergotamina es un tratamiento abortivo eficaz para migraa, pero es un vasoconstrictor y no debe usarse en pacientes con angina de pecho. En la valoracin de ancianos con cefalea de inicio reciente debe considerarse realizar una tomografa computadorizada del cerebro. En este caso, si es posible suprimir la cefalea al suspender la nitroglicerina, no es necesaria una CT.

11. Masculino 55 aos que ha fumado 30 cigarrillos al da desde que tena 25 aos acude a sala de urgencias presentando hemoptisis. Refiere tos que produce 5 a 10 ml. de esputo cada maana. Los resultados de la exploracin fsica y la radiografa de trax son normales. Causa de la hemoptisis en este enfermo? (A) Carcinoma broncgeno (B) Tuberculosis pulmonar (C) Bronquiectasias (D) Bronquitis crnica .- Allen R. M. MMS Medicina Interna. 5. Edicin. National Medical Series. Mc. Graw Hill. 2006. (captulo 2 IIA 1 a, E 2 a (2); IV A 4 c). Por definicin (tos con produccin de esputo), este paciente tiene bronquitis crnica. Los resultados normales de la radiografa de trax no descartan absolutamente un carcinoma, pero s indican que este diagnstico es poco probable. Lo mismo es cierto para la tuberculosis y la bronquiectasia. Un fumador de este grupo de edad requiere broncoscopia si la hemoptisis no cede pronto o si la situacin clnica cambia. La deficiencia de antitripsina a, es un factor gentico que predispone a sufrir enfisema. A diferencia de la bronquitis crnica, el enfisema se relaciona con poca tos y expectoracin o ninguna. 12. La siguiente aseveracin es correcta respecto a la hipertensin reno vascular? (A) Se relaciona con mayor liberacin de renina (B) No mejora con el tratamiento con captopril (C) Se ve a menudo en jvenes como complicacin de enfermedad fibromuscular

(D) Es fcil de controlar

Allen R. M. MMS Medicina Interna. 5. Edicin. National Medical Series. Mc. Graw Hill. 2006. (captulo 6, parte I: XIV C, parte III: II B 1). La hipertensin renovascular se relaciona con un incremento en la liberacin de renina. La disminucin del flujo sanguneo renal estimula la liberacin renal de cantidades en aumento de renina, que activan el sistema reninaangiotensina-aldosterona y producen hipertensin. El captopril impide la conversin de angiotensina I a angiotensina II (un vasoconstrictor que se esperara fuera de especial eficacia en esta enfermedad). Las mujeres jvenes, mucho ms a menudo que los varones jvenes, presentan obstruccin fibromuscular de las arterias renales, lo que provoca hipertensin reno-vascular. Tpicamente sta causa hipertensin muy grave y es notablemente difcil de controlar; este hecho hace sospechar que hay hipertensin renovascular.

13. Fem. 45 aos tuvo una cifra al azar de glucosa de 180 mg/100 dl. El Examen medico de eleccin para esta paciente es? (A) Glucosa en orina (B) Prueba de tolerancia oral a la glucosa (C) Glucosa plasmtica en ayunas (D) Repetir la medicin de glucemia al azar Allen R. M. MMS Medicina Interna. 5. Edicin. National Medical Series. Mc. Graw Hill. 2006. (captulo 9 IV A 5, cuadro 9-10). Aunque la prueba oral de tolerancia a la glucosa (glucemia 2 h despus de una carga de 175 g de glucosa) es ms sensible, se recomienda la glucemia en ayunas como prueba inicial para diagnosticar diabetes mellitus, debido a su mayor conveniencia. Una glucemia en ayunas de 126 mg/100 mi o mayor es diagnstica de diabetes. Aunque una cifra de glucemia aleatoria mayor de 200 mg/100 mi indica diabetes en un paciente con sntomas clsicos como poliuria y polidipsia, no se recomienda para investigacin. La medicin de HbA,c y en ocasiones de glucosa en orina es til para valorar el control de la glucemia en diabticos, pero no son lo bastante confiables o estn estandarizadas para ser de gran utilidad en el diagnstico de diabetes.

14. Fem. 65 aos presenta dolor, calor y tumefaccin en la rodilla derecha. Dato til para establecer el diagnstico de pseudogota en esta paciente? (A) Crecimiento de articulaciones interfalngicas proximales e interfalngicas distales (B) Altas concentraciones sricas de cido rico (C) Cristales con birrefringencia negativa del lquido de la rodilla (D) Calcio en el menisco en una radiografa de la rodilla afectada

Allen R. M. MMS Medicina Interna. 5. Edicin. National Medical Series. Mc. Graw Hill. 2006. (captulo 10 IV B 5 b). El hallazgo de un menisco calcificado en la radiografa de la rodilla

afectada es un dato diagnstico de enfermedad por dihidrato de pirofosfato de calcio, que sugiere que la inflamacin de la rodilla es causada por seudogota. El crecimiento de las articulaciones interfalngicas proximales y distales sugiere slo osteoartritis y no una causa especfica. El aumento srico de urato se relaciona con gota. El aumento de cristales con birrefringencia en examen de luz polarizada compensada roja, de lquido sinovial, es especfico para el diagnstico de seudogota. Hay muchas causas de derrame inflamatorio aparte de esta entidad.

15. Fem. 25 aos visita a su mdico familiar por obstruccin nasal y rinorrea. Antecedente de utilizar descongestivo nasal en atomizador en un episodio reciente que se exacerbo con clima hmedo. En la rinoscopia anterior, las cutirreacciones y el frotis citolgico nasal no muestran infeccin o anomala anatmica o inmunitaria. El diagnostico de esta paciente es? (A) Abuso de descongestivo nasal en atomizador (B) Rinitis alrgica (C) Rinitis no alrgico eosinoflica (D) Rinitis vasomotora

Allen R. M. MMS Medicina Interna. 5. Edicin. National Medical Series. Mc. Graw Hill. 2006. (captulo 7IIIE 2 a-h). Es probable que los sntomas de esta mujer se deban a rinitis vasomotora, sndrome caracterizado por obstruccin nasal y rinorrea sin datos de enfermedad inmunitaria o infecciosa nasal. Con frecuencia, los sntomas de rinitis vasomotora empeoran despus que los pacientes sufren tensin emocional o experimentan un cambio en la temperatura corporal o ambiental, en la posicin corporal o clima hmedo. La rinitis vasomotora no mejora con medicamentos. Los antecedentes de este enfermo no indican abuso de descongestivos en atomizadores. El interrogatorio y la exploracin tambin descartan embarazo y desviacin del tabique, ambas posibles causas de rinorrea y obstruccin nasal. La rinitis alrgica, que es un trastorno mediado por IgE, se caracteriza por mucosa nasal plida, abotagada, eosinofilia nasal y cuti-rreacciones positivas. Aunque la obstruccin nasal y la rinorrea son sntomas de rinitis eosinfila no alrgica, este trastorno se caracteriza por eosinofilia nasal pronunciada, que se hubiera identificado en el frotis nasal. Los plipos nasales son otros sntomas de rinitis no alrgica eosinfila. No es probable que los sntomas relacionados con polipsis nasal sean episdicos. 16. Masculino 18 aos presenta cefalea persistente y fiebre, y despus de cinco das, un cuadro de crisis convulsiva tnica focal. La TAC de crneo muestra una lesin con reforzamiento anular en el lbulo frontal derecho y un nivel hidroareo en el seno frontal. Se realiza aspirado neuroquirrgico de la lesin, Cul seria el hallazgo en este paciente? (A) Clulas mononucleares pequeas sugestivas de linfoma de Burkitt (B) Quistes de Toxoplasma gondii y taquizotos (trofozotos) (C) Escherichia coli y Bacteroides fragilis (D) Estreptococo hemoltico a y mezcla de anaerobios

Allen R. M. MMS Medicina Interna. 5. Edicin. National Medical Series. Mc. Graw Hill. 2006. (Captulo 8 V A 3 a, b). El aspirado neuroquirrgico de esta lesin probablemente muestre estreptococos hemolticos a y anaerobios mezclados. Los abscesos cerebrales bien pueden presentarse en adolescentes. La enfermedad del lbulo frontal suele coexistir con sinusitis e indica flora de la boca. En abscesos de origen tico es ms comn encontrar Escherichia coli y Bacteroidesfragilis. El linfoma cerebral primario y la toxoplasmosis se observan rara vez en individuos inmunocompetentes. En personas con acidosis grave los cigomicetos pueden causar absceso cerebral, pero estos hongos no tienen fase de levadura. 17. Fem. 20 aos se encuentra hospitalizada con el siguiente hallazgo trombosis venosa profunda de nueva aparicin izquierda. Antecedentes de trombocitopenia leve y dos abortos; fue tratada para sfilis hace dos aos debido a una prueba de reagina rpida del plasma positiva, aunque el anticuerpo antitreponmico fue negativo. De los siguientes trastornos el que represente el conjunto de su historia clnica es? (A) Sndrome de anticuerpos antifosfolpido (B) Lupus eritematoso diseminado (SLE) (C) Sndrome de anticuerpo Ro (D) Arteritis de Takayasu Allen R. M. MMS Medicina Interna. 5. Edicin. National Medical Series. Mc. Graw Hill. 2006. (captulo 10VIIFl, 2c, G3b). Esta paciente tiene varias manifestaciones sutiles de sndrome de anticuerpo antifosfolpido, que en conjunto hacen de ste un diagnstico probable. La prueba de reagina plasmtica rpida (RPR) positiva, con resultados negativos de la prueba de treponemas, probablemente refleja anticuerpos que producen reaccin cruzada a la cardiolipina o a componentes fosfolpidos de antgenos treponmicos. La trombocitopenia es frecuente en esta situacin, debido a las interacciones de plaquetas y clulas endoteliales y a la coagulacin inducida por los anticuerpos. Los abortos pueden deberse a coagulacin en vasos placentarios pequeos. Las trombosis venosas profundas o incluso la coagulacin en arterias mayores pueden ser causadas por hipercoagulabilidad consecuente. No hay otras pruebas de lupus eritematoso diseminado, aunque pueden encontrarse anticuerpos antifosfolpido en alrededor de 33% de los pacientes. Tampoco se describen datos de sndrome de anticuerpo Ro o enfermedad indiferenciada del tejido conjuntivo. La arteritis de Takayasu se presenta en mujeres asiticas jvenes, pero no hay datos de isquemia de grandes vasos arteriales caracterstica de esta enfermedad.

18. Jardinero 60 aos de edad presenta cambios de personalidad. Su familia indica que varias semanas antes se quej de dolor en la mueca y hace una semana tuvo asimetra facial transitoria. Sera un dato importante a considerar? (A) Exposicin a toxinas (B) Hbitos sexuales (C) Picadura de garrapata

(D) Picadura de mosquito

Allen R. M. MMS Medicina Interna. 5. Edicin. National Medical Series. Mc. Graw Hill. 2006. (captulo 8 VII F 3 b, c). La enfermedad de Lyme se transmite por picadura de garrapata. Las complicaciones neurolgicas son neuropata craneal, radiculopata y encefalopata. Las actividades en el exterior de este paciente lo ponen en riesgo de exposicin a enfermedad de Lyme si vive en un rea endmica y la sugerencia de artritis de la mueca concuerda con las manifestaciones sistmicas de aqulla. La exposicin a toxinas puede causar confusin, pero no produce artritis o debilidad facial. Una conducta sexual de alto riesgo predispone a infeccin por virus de la inmunodeficiencia humana (VIH) que puede causar parlisis de nervio facial y encefalopata. Sin embargo, dados los antecedentes y la presentacin de este enfermo, es ms probable la enfermedad de Lyme. Una picadura de mosquito puede transmitir encefalitis vrica. Sin embargo, no hay antecedente de fiebre y la artritis de la mueca y la debilidad facial no apoyan un diagnstico de encefalitis vrica temprana. El consumo excesivo de vitamina no produce artritis, neuropata de pares craneales ni cambios de la personalidad. 19. Fem. 40 aos goza de buena salud experimenta dolor retroesternal sbito con fiebre y falta de aire. Es fumadora y no toma medicamentos excepto anticonceptivos orales. En la exploracin fsica se encuentran taquipnea y temperatura de 38C. Los datos de auscultacin, percusin y radiogrficos del trax son normales. Diagnstico probable en esta paciente? (A) (B) (C) (D) Traqueobronquitis Neumona atpica Embolia pulmonar Neumona bacteriana

Allen R. M. MMS Medicina Interna. 5. Edicin. National Medical Series. Mc. Graw Hill. 2006. (captulo 2 VIII E 1, 2 a; captulo 8 V C 3, 4). El diagnstico ms probable es embolia pulmonar. El inicio agudo descarta neumona atpica y hace poco probable el cncer pulmonar. Sin pruebas de tos productiva es poco probable que haya traqueobronquitis, trastorno tambin subagudo. La neumona bacteriana es muy improbable junto con la radiografa de trax. El tabaquismo y el uso de anticonceptivos orales predisponen a trombosis venosa profunda y embolias pulmonares. 20. Fem. 25 aos de edad presenta fiebre y artritis inflamatoria que afecta a las articulaciones metacarpofalngicas e interfalngicas proximales Diagnostico diferencial entre lupus eritematoso diseminado y artritis reumatoide? (A) Sedimento urinario activo (eritrocitos, leucocitos, cilindros celulares, sin bacterias) (B) Artritis inflamatoria de articulaciones metacarpofalngicas e interfalngicas (C) Derrame pleural de La radiografa de trax (D) Anemia

10

Allen R. M. MMS Medicina Interna. 5. Edicin. National Medical Series. Mc. Graw Hill. 2006. (captulo 10IIG; VIIG l;cuadro 10-10). Un sedimento urinario activo sugiere glomerulonefritis, dato comn en pacientes con lupus eritematoso diseminado, pero no en quienes tienen artritis reumatoide. Pueden encontrarse artritis de articulaciones metacarpofalngicas e interfalngicas proximales derrames pleurales y anemia en cualquiera de las dos enfermedades. Las anomalas de la funcin heptica son atpicas en el lupus eritematoso sistmico, pero comnmente son resultado de disfuncin heptica relacionada con frmacos (p. ej., por antiinflamatorios no esteroideos en cualquier enfermedad. 21. Un paciente con carcinoma pulmonar presenta nuseas, vmitos y letargo y se encuentra que tiene una concentracin srica de calcio de 13.4 mg/100 ml. Cul debe ser el primer paso para su tratamiento? (A) Etidronato intravenoso (B) Mitramicina intravenosa (C) Glucocorticoides intravenosos (D) Solucin salina y furosemida intravenosos

Allen R. M. MMS Medicina Interna. 5. Edicin. National Medical Series. Mc. Graw Hill. 2006. (captulo 9 IIIA 6 a, b, d, 7 c (1), (3)). La hipercalciemia causada por otras enfermedades aparte del hiperparatiroidismo puede tratarse con solucin salina y furosemida intravenosos. El reemplazo de lquidos con solucin salina intravenosa, seguido por diuresis forzada con solucin salina y furosemida intravenosas, es una forma rpida y segura de disminuir el calcio srico y debe intentarse primero. Pueden aadirse pamidronato, mitramicina o calcitonina si se necesita disminuir an ms la concentracin de calcio. Los glucocorticoides son eficaces para tratar la hipercalciemia causada por exceso de vitamina D, sarcoidosis y algunas neoplasias malignas hemticas, pero no disminuyen el calcio srico en la mayor parte de los casos de hipercalciemia relacionada con tumores slidos.

22. Fem. De 27 aos llega al servicio de urgencias con disnea y dolor torcico. Refiere que en los cuatro das previos tuvo tumefaccin e hipersensibilidad en la pantorrilla y el muslo derechos. Con base en la presentacin clnica se sospecha una trombosis venosa profunda que podra haber causado embolia pulmonar. Cul de los siguientes fragmentos de informacin de los antecedentes de la enferma apoyaran mejor este diagnstico? (A) Antecedente de tabaquismo (B) Antecedente de diabetes mellitus en la familia de la paciente (C) Antecedente de lesin en la extremidad inferior (D) Antecedente de hipertensin

Allen R. M. MMS Medicina Interna. 5. Edicin. National Medical Series. Mc. Graw Hill. 2006. (captulo 1 VIIIA 2 d, 3 a) La lesin de las extremidades inferiores puede producir formacin de cogulos sanguneos y desarrollo de tromboflebitis. Otros factores que contribuyen al

11

desarrollo de trombosis venosa profunda incluyen uso de compuestos con estrgenos (p. ej., anticonceptivos orales) o inmovilizacin de la extremidad inferior (p. ej., durante ciruga o reposo en cama prolongado) que da lugar a estasis venosa. La hipertensin, la diabetes mellitus y el abuso de drogas intravenosas no tienen relacin con la trombosis venosa profunda. La disnea y el dolor pleurtico sugieren que la trombosis venosa profunda ha causado embolia pulmonar, que es provocada por desplazamiento de un trombo de las venas de las extremidades inferiores o la pelvis a la arteria pulmonar. 23. Establece el diagnstico de trombosis venosa profunda? (A) Cateterismo cardaco (B) Venografa con contraste (C) Pletismografa de impedancia (D) Estudios de ventilacin y perfusin pulmonares

Allen R. M. MMS Medicina Interna. 5. Edicin. National Medical Series. Mc. Graw Hill. 2006. (captulo 1 VIIIA 4 a-b), La pletismografa de las extremidades inferiores es una prueba incruenta til para establecer el diagnstico de trombosis venosa profunda. La venografa con medio de contraste proporciona el diagnstico definitivo casi en cualquier caso; sin embargo, esta prueba cruenta puede, de hecho, causar tromboflebitis en una minora de los casos. El cateterismo cardaco, el gammagrama pulmonar y la tomografa computadorizada (CT) no son tiles para diagnosticar trombosis venosa profunda. 24. El diagnstico de embolia pulmonar se hace con el uso de: (A) Gasometra arterial (B) Radiografa torcica (C) Electrocardiograma (ECG) (D) Arteriografa pulmonar Allen R. M. MMS Medicina Interna. 5. Edicin. National Medical Series. Mc. Graw Hill. 2006. (captulo 2 VIII E 6 a-c, 7, 8 a). La prueba ms sensible y especfica de embolia pulmonar es la arteriografa pulmonar. El gammagrama nuclear pulmonar es otra tcnica til, pero no es tan especfico como la arteriografa. Aunque un gammagrama normal casi descarta embolia pulmonar, los resultados a menudo caen en el lmite de probabilidad intermedio, lo que dificulta establecer un diagnstico definitivo. En la mayor parte de los casos de embolias pulmonares, el electrocardiograma (ECG) es normal. La desviacin aguda del eje a la derecha observado en ECG puede producir diagnstico errneo de infarto miocrdico anterior (MI). La hipoxia, la hipocapnia y la alcalosis respiratoria son datos clsicos de la gasometra, pero son inespecficos de embolia pulmonar. La radiografa de trax es normal, sobre todo si no ha habido infarto. 25. Un varn de 42 aos con antecedente de trastornos convulsivos experimenta una crisis de gran mal. Sus pruebas de laboratorio, tomadas poco despus, revelan lo siguiente: sodio srico a 140 meq/L; potasio srico, 4.1 meq/L; cloruro srico, 97 meq/L; bicarbonato

12

plasmtico (HCO3-), 16 meq/L; pH arterial, 7.15 y Paco2, 46 mmHg. Cul de los siguientes cuadros describe la alteracin acidobsica? (A) Acidosis respiratoria (B) Acidosis metablica (C) Acidosis metablica ms acidosis respiratoria (D) Acidosis respiratoria ms alcalosis respiratoria

Allen R. M. MMS Medicina Interna. 5. Edicin. National Medical Series. Mc. Graw Hill. 2006. (captulo 6, parte II: IV B-D). El paciente tiene acidemia importante, (bajo pH arterial), relacionada con bajo Hco3- srico; por tanto, debe haber acidosis metablica. Este trastorno se debe a acumulacin de lactato debida a actividad convulsiva. Otro dato es la gran brecha aninica. Adems, el paciente tambin tiene acidosis respiratoria, como se manifiesta por PaCO2 alta. Con frecuencia, la hiperventilacin acompaa a convulsiones de gran mal. 26.Masculino 18 aos de edad, acude a su clnica un lunes por la maana, el motivo de consulta es la presencia de ms de 8 evacuaciones lquidas con sangre, moco y pujo, las ltimas 24 horas, precedidas de dolor tipo clico que cede posterior a la defecacin, adems de tenesmo rectal. Llama la atencin que este cuadro inicio hace dos semanas, al principio solo existi disminucin de la consistencia de la materia fecal, hasta que, alrededor del cuarto da, solo haba evacuaciones lquidas. Afebril y sin vmitos. La exploracin fsica demuestra mucosas orales humectadas. Dolor a la palpacin de todo el abdomen, sin rebote, con ruidos intestinales intensos y abundantes. Su presin arterial es de 100/75. Es el diagnstico en este caso es? (A) Diarrea aguda. (B) Diarrea crnica. (C) Colitis por parsitos. (D) Enfermedad intestinal inflamatoria.

Instituto Nacional de la Nutricin. Salvador Zubiran Manual de teraputica mdica y procedimientos de urgencias. Cuarta edicin. Pag. 443 450. Mac Graw-Hill Interamericana. Mxico. La diarrea aguda se define como una alteracin sbita del hbito intestinal. Se caracteriza por aumento de la frecuencia (>3/da) y disminucin de la consistencia de las evacuaciones, la cual persiste durante menos de tres semanas. De manera ms objetiva, se considera diarrea cuando las heces de 24 horas contienen ms de 200 g. de agua. 27. Usted solicita un frotis fecal en el cual encuentra leucocitos abundantes, adems de eritrocitos, por lo que, con estos datos ms el cuadro clnico descrito en la pregunta nmero 26, la clasificara cmo: (A) Colitis inmunolgica (B) Diarrea aguda inflamatoria (C) Diarrea aguda no inflamatoria (D) Enfermedad intestinal inflamatoria.

13

Instituto Nacional de la Nutricin. Salvador Zubiran Manual de terapetica mdica y procedimientos de urgencias. Cuarta edicin. Pag. 443 450. Mac Graw-Hill Interamericana. Mxico. Existen diversas clasificaciones de la diarrea aguda; sin embargo, desde el punto de vista clnico, una de las ms tiles es diferenciarla en inflamatoria (con dao a la mucosa) y no inflamatoria (sin dao a la mucosa). Las del tipo inflamatorio por lo general son causadas por bacterias invasoras o productoras de citotoxinas, o por parsitos que afectan de preferencia al colon. En cambio, las de tipo no inflamatorio de ordinario son causadas por virus o bacterias productoras de enterotoxinas que afectan en particular al intestino delgado. 28. Otitis que se acompaa con secrecin y signos locales, sistmicos, o ambos, debe ser tratada con: (A) Antihistamnicos de forma exclusiva (B) Solo medidas generales (C) Drenaje, AINES y esteroides locales (D) Antibiticos, antihistamnicos y AINES

Instituto Nacional de la Nutricin. Salvador Zubiran Manual de teraputica mdica y procedimientos de urgencias. Cuarta edicin. Pag. 443 450. Mac Graw-Hill Interamericana. Mxico. La otitis media se define como la inflamacin del odo medio que se acompaa de secrecin y signos locales, sistmicos, o ambos, de enfermedad aguda. Esta enfermedad predomina en la infancia, sin embargo tambin existe en el adulto. Su distribucin tiene una clara periodicidad estacional, sobre todo en otoo e invierno. Los agentes de mayor prevalencia en nuestro pas son: Streptococcus pneumoniae, Haemophilus influenzae, Morexella catarrhalis estreptococo del grupo A y S. aureuse. Los antibiticos ms aceptados en estos casos son la amoxicilina, la amoxicilina con clavulanato, la calritromicina y el trimetropim con slfametoxazol. Se acepta el uso de antihistamnicos y antiinflamatorios no esteroideos para disminuir la congestin de la mucosa para resolver la obstruccin de la trompa farngotimpnica; en realidad no se ha demostrado efecto sobre la duracin de los sntomas. El drenaje del derrame solo esta indicado cuando ste persiste por ms de tres semanas. 29. Un empleado de hospital de 27 aos de edad presenta induracin en el sitio de prueba con derivado de protena purificado (PPD) realizado como parte de un examen de muestreo sistemtico. Dato que contraindica la quimioprofilaxis con isoniazida? (A) Induracin de 4 mm de dimetro (B) Cutirreaccin con PPD negativa un ao antes (C) Cutirreaccin positiva un ao antes (D) Exposicin extensa reciente a un vecino con tuberculosis activa Allen R. M. MMS Medicina Interna. 5. Edicin. National Medical Series. Mc. Graw Hill. 2006. (captulo 8 VII C 3). Una induracin de 4 mm de dimetro en el sitio de prueba con derivado de protena purificado (PPD) va contra el uso de quimioprofilaxis con isoniazida. A todos los

14

adultos menores de 35 aos con prueba PPD positiva se les realiza quimioprofilaxis con isoniazida a no ser que no puedan tolerarla y ya hayan recibido tratamiento completo, o se sepa que han estado expuestos a tuberculosis resistente a isoniazida. Se requiere una induracin de por lo menos 10 mm para considerar una prueba positiva. Los grados menores de induracin pueden representar reacciones cruzadas a otras micobacterias. 30. Fem. 66 aos presenta deterioro al caminar. En la exploracin se encuentra marcha ligeramente espstica, mala posicin y sensacin de vibracin en los dedos de los pies, reflejos de estiramiento muscular en las rodillas +++ y reflejos aquleos ausentes. El diagnstico probable es? (A) Esclerosis mltiple (B) Hipovitaminosis B12 (C) Hidrocefalia con presin normal (NPH) (D) Infeccin con virus linfotrpico humano de clulas T tipo I (HTLV-I)

Allen R. M. MMS Medicina Interna. 5. Edicin. National Medical Series. Mc. Graw Hill. 2006. (captulo 11III B 4 a; XI A, XIX A 2 a). La hipovitaminosis B12 causa degeneracin combinada de sistemas. Los pacientes presentan alteracin de la marcha caracterizada por espasticidad y disminucin de la sensacin vibratoria y la posicin. Puede haber neuropata leve que produce depresin de los reflejos aquleos. Como es una causa tratable de marcha anormal, es importante identificar la deficiencia de vitan B12. La esclerosis mltiple (MS) no se manifiesta en este grupo de edad. La ausencia de reflejos aquleos es un dato comn de MS; ms bien, la hiperactividad de los reflejos concuerda con los datos de neurona motora superior encontrados a menudo. La hidrocefalia de presin normal (NPH) es una causa de deterioro de la marcha. Sin embargo, hay donacin cognitiva y los pacientes pueden tener incontinencia urinaria. La alteracin de la vibracin y la sensacin de posicin, as como la ausencia de reflejos aquleos, no son caractersticos de NPH. La infeccin virus linfotrpico de clulas T humano tipo I (HTLV-I) es causa de mielopata y debe sospecharse en pacientes que han recibido transfusiones de sangre, usuarios de drogas intravenosas o personas que hayan habitado en reas endmicas. La adrenomieloneuropata es un trastorno recesivo ligado a X que guarda relacin con adrenoleucodistrofia, la cual tpicamente se presenta en varones jvenes. El trastorno origina la acumulacin de cidos grasos de cadena muy larga debido a una imposibilidad para catabolizar estos lpidos. Las mujeres portadoras manifiestan una paraparesia espstica leve, pero no la gama de datos que se encuentran en esta paciente. 31. Cul de las siguientes aseveraciones describe mejor a los pacientes con apnea obstructiva durante el sueo? (A) Tienen diferentes presentaciones clnicas que los sujetos con otras formas de apnea durante el sueo (B) Responden bien al tratamiento con estimulantes respiratorios (C) Responden bien a la presin positiva continua en las vas nasales (D) No tienden a quedarse dormidos durante el da

15

Allen R. M. MMS Medicina Interna. 5. Edicin. National Medical Series. Mc. Graw Hill. 2006. (captulo 2 XV B-E). Los pacientes con apnea obstructiva evolucionan bien con pre positiva continua por va nasal (CPAP), que acta como frula de la parte posterior obstruida de la faringe personas con apnea durante el sueo, ya sea central, obstructiva, o mixta, son indistinguibles desde el pi de vista clnico; se requieren estudios del sueo para averiguar qu tipo de apnea est presente. Aun cuado algunos pacientes con apnea obstructiva pueden mejorar con estimulantes respiratorios, otros requieren m das ms drsticas. Los enfermos con todas las formas de apnea tienen somnolencia durante el da y, por razones desconocidas, estn en riesgo de sufrir cor pulmonale. 32. Masculino 30 aos presenta dolor y tumefaccin del testculo derecho. Su mdico solicita un ultrasonido que revela una masa testicular de 2 x 2.5 cm. Se realizan una exploracin inguinal y una orquiectoma. El estudio histopatolgico revel un seminoma puro. Una tomografa computadorizada de trax, abdomen y pelvis mostr dos ganglios retroperitoneales de 3 cm que estn aumentados de tamao. La biometra hemtica, la qumica sangunea y los marcadores tumorales estn todos dentro de los lmites normales. Conducta a seguir en este paciente es? (A) Extirpacin quirrgica de toda la enfermedad (B) Radioterapia. (C) Quimioterapia (D) Braquiterapia Allen R. M. MMS Medicina Interna. 5. Edicin. National Medical Series. Mc. Graw Hill. 2006. (captulo 4IX G 2). El cncer testicular es el ms comn en varones adultos jvenes variedades ms frecuentes son seminomas y tumores de clulas germinales no seminomatosos; ambos sor rabies aun en etapas avanzadas. Los seminomas son muy sensibles a la radioterapia; por tanto, los pacientes enfermedad de etapa II (la limitada a testculo y a ganglios por abajo del diafragma) pueden tratarse con < bajas de radiacin. Debido a la toxicidad de la mdula sea producida por la radioterapia mediastnica, y est indicada la radiacin profilctica del mediastino. En este contexto, los ganglios linfticos retroperito les aumentados de tamao denotan la existencia de enfermedad metastsica y est indicado el tratamienl 33. De los siguientes pacientes con artritis reumatoide el que se encuentra con mayor riesgo de complicarse en artritis crnica, erosiva e incapacitante? (A) Pacientes con oligoartritis y afeccin de la columna axial (B) Persona con oligoartritis sin afeccin de la columna axial (C) Enfermo con artritis reumatoide juvenil de inicio sistmico (D) Individuo con artritis poliarticular seropositiva para factor reumatoide

Allen R. M. MMS Medicina Interna. 5. Edicin. National Medical Series. Mc. Graw Hill. 2006. (captulo 10XIIE2c (l), H2a).El subgrupo de pacientes con artritis reumatoide juvenil que con ms probabilidad desarrolle artritis crnica erosiva y grave es el de grupo poli articular seropo

16

sitivo para factor reumatoide. Esta forma de enfermedad tambin es la ms parecida a artritis reumatoide de adulto y es ms probable que requiera un agente de segunda lnea. 34. Fem. 42 aos experiment recientemente fatiga, somnolencia, piel seca, estreimiento y aumento de peso de 5 kg. Su tiroides est firme y tiene el doble del tamao normal. Confirma el diagnstico de hipotiroidismo? (A) Tiroxina srica (T4) (B) Triyodotironina srica (T3) (C) Captacin de resina T3 (D) Hormona estimulante de tiroides (TSH) en suero

Allen R. M. MMS Medicina Interna. 5. Edicin. National Medical Series. Mc. Graw Hill. 2006. (captulo 9 II A 5). Lo ms probable es que la medicin de hormona estimulante de tiroides (TSH) en suero confirme un presunto diagnstico de hipotiroidismo. En un caso leve de hipotiroidismo primario, por lo comn TSH alcanza concentraciones anormales antes que la tiroxina (T4) o la triyodotironina (T3) sricas hayan descendido por abajo de los lmites normales. La captacin de resina de T3, que no es una prueba de alta precisin del funcionamiento tiroideo, se usa sobre todo para descartar anomalas de las protenas de unin a hormona tiroidea. Un alto ttulo de anticuerpos antitiroideos indica la presencia de tiroiditis crnica, que es la causa ms comn de hipotiroidismo, pero no indica si hay hipotiroidismo. 35. Masculino 71 aos presenta gota de inicio agudo. Este problema ha recurrido durante varios aos y por lo comn se manifiesta como artritis monoarticular aguda que afecta la primera articulacin intertarsal proximal metatarsiana. Tambin tiene antecedentes de larga duracin de insuficiencia renal crnica con cifras de creatinina srica de 4 a 6 mg/100 mi en los ltimos cinco aos e hipertensin de mucho tiempo antes tratada con diversos productos como diurticos y bloqueadores adrenrgicos. En la exploracin fsica su presin sangunea es de 170/105 mmHg, su pulso es de 72/min, su frecuencia respiratoria de 15/min y su temperatura de 37C. En la exploracin subsiguiente se encuentra cardiomegalia moderada con galope con tercer ruido cardaco (S3) y cuarto ruido cardaco (S4), as como tumefaccin e hipersensibilidad de la primera articulacin metatarsiana derecha. En los estudios de laboratorio se encuentra lo siguiente: nitrgeno ureico sanguneo (BUN) de 63 mg/100 mi; creatinina de 5.1 mg/100 mi; sodio srico de 136 meq/L; potasio srico de 5.9 meq/L; cloruro srico de 100 meq/L; C02 de 19 meq/L y cido rico de 9.3 mg/100 mi. La causa probable del trastorno de este paciente es? (A) Nefropata crnica por plomo (B) Exceso de produccin primaria de cido rico (C) Nefritis intersticial crnica relacionada con abuso de analgsicos (D) Nefropata hipertensiva

Allen R. M. MMS Medicina Interna. 5. Edicin. National Medical Series. Mc. Graw Hill. 2006. (captulo 6, parte I: XII B 2 b). La nefropata crnica por plomo es la causa ms probable del

17

trastorno de este paciente; coexiste con afeccin de la excrecin de cido rico y tambin con sndrome clnico de gota. Si bien sta y la insuficiencia renal crnica de otras causas pueden coexistir, las prueba; actuales sugieren que la toxicidad por plomo es la causa de gran parte de los casos en que concurren ambas enfermedades. El abuso de analgsicos no debe relacionarse con una mayor incidencia de gota, aunque los dato: renales pueden ser semejantes. La nefropata hipertensiva tambin puede explicar los datos, aunque no la aparicin de gota en este paciente. Podra haber hipertensin renovascular en este enfermo, pero se relaciona con hipertensin ms grave cuando hay insuficiencia renal. Adems, aqulla coexiste con gota. Sin embargo, se necesitan imgenes del rbol vascular renal para descartar por completo este diagnstico. 36. Fem. 28 aos con antecedente de migraa se presenta con cefalea constante de tipo compresivo relacionada con fotofobia y temperatura de 39.1C. Por medio de puncin lumbar se extrajo lquido cefalorraqudeo con una concentracin de protenas de 62 mg/100 mi, glucosa de 76 mg/100 mi y 26 mononucleares/mm3. Dos das despus la paciente se encuentra afebril y slo tiene cefalea al levantarse Diagnstico probable de esta persona? (A) Migraa (B) Meningitis bacteriana (C) Meningitis asptica (D) Cefalea postural posterior a puncin lumbar

Allen R. M. MMS Medicina Interna. 5. Edicin. National Medical Series. Mc. Graw Hill. 2006. (captulo 8 VA 1 a (1) (a)-(b); captulo 11IV C 2). Lo ms probable es que esta enferma haya tenido meningitis asptica de resolucin espontnea y sufra cefalea postural posterior a puncin lumbar (LP) al despertarse. Otros sntomas pueden ser nuseas, visin borrosa, acfenos y vmitos. La disminucin de la presin de lquido cefalorraqudeo (LCR) puede ser la causa de cefalea relacionada con puncin lumbar Las opciones teraputicas son reposo en cama, analgsicos y parche sanguneo epidural lumbar que en parte puede taponar un desgarro en la duramadre. Aunque la paciente tiene antecedente de migraa, es inusual que las cefaleas vasculares se presenten slo en bipedestacin. La meningitis bacteriana es una complicacin rara de LP. Sin embargo, como la paciente est afebril y slo ha tenido cefalea en posicin erecta, este diagnstico es improbable. Del mismo modo lo es que la cefalea se deba a meningitis asptica persistente. Los enfermos pueden presentar hipersensibilidad y tensin de los msculos cervicales despus de meningitis asptica. Esto puede confundirse con meningismo persistente. Sin embargo, es ms probable que esta persona sufra cefalea como resultado de complicaciones de LP.

37. Fem. 17 aos tiene exantema cutneo rojo difuso; fiebre de 39.4C y diarrea leve acuosa. En fechas recientes tuvo infeccin de garganta por la que se le administr sulfametoxazol. Comenz sus menstruaciones hace tres das. En la exploracin fsica se encuentran cambios eritematosos difusos de la piel con descamacin temprana. La boca y las conjuntivas estn eritematosas. Cul de los siguientes entidades explica todo el proceso?

18

(A) Bacteriemia por Salmonella (B) Sndrome de choque txico (TSS) (c) Tuberculosis (D) Mononucleosis por virus de Epstein-Barr Allen R. M. MMS Medicina Interna. 5. Edicin. National Medical Series. Mc. Graw Hill. 2006. (captulo 8 VII D 1 a, 2 b). La bacteriemia por Salmonella, el sndrome de choque txico (TSS), la tuberculosis y la mononucleosis de Epstein-Barr pueden acompaarse de fiebre, pero la presencia de exantema descamativo difuso sugiere TSS, reaccin farmacolgica grave (p. ej., sndrome de Stevens-John-son); enfermedad de Kawasaki, o escarlatina. El exantema cutneo relacionado con salmonelosis es muy sutil y evanescente (manchas de color rosa). La tuberculosis no se caracteriza por afeccin cutnea difusa y de mucosas o diarrea acuosa. Si bien la alergia al sulfametoxazol puede producir eritema cutneo y de mucosas, no causa diarrea. 38. Femenino 24 aos refiere que sus manos se tornan blancas y luego azules en el fro. Que dato sugiere esclerodermia como causa de sndrome de Raynaud en la paciente? (A) Engrosamiento cutneo distal que se extiende en direccin proximal hasta las articulaciones metacarpofalngicas (B) Anticuerpos anticentrmero en suero (C) Anticuerpos antinucleares en suero (D) Cambios capilares distales en la valoracin del lecho ungueal

Allen R. M. MMS Medicina Interna. 5. Edicin. National Medical Series. Mc. Graw Hill. 2006. (captulo 10 VIII F 1). El engrosamiento de la piel es una caracterstica definitoria de esclerodermia; si se limitan las reas distales puede ser manifestacin de la variante CREST de esclerodermia (esclerodermia que coexiste con calcinosis subcutnea, fenmeno de Raynaud, disfuncin de la motilidad esofgica, esclerodactilia y telangiectasia). Los pacientes con fenmeno de Raynaud seropositivos para anticuerpos antinucleares (ANA) o anticuerpos anticentrmero o que muestran cambios capilares digitales, estn en ms alto riesgo de presentar esclerodermia que un paciente con fenmeno de Raynaud sin esas caractersticas. La dismotilidad, el espasmo o la estenosis esofgicos distales son posibles datos de esclerodermia, pero no son especficos. 39. Masculino 73 aos acude a una cena familiar de celebracin y pasa una gran noche. Al dejar el restaurante sufre un colapso. Cul de los siguientes trastornos es probable? (A) Sncope posprandial (B) Arritmia cardaca (C) Hipersensibilidad del seno carotdeo (D) Quiste coloide del tercer ventrculo

19

Allen R. M. MMS Medicina Interna. 5. Edicin. National Medical Series. Mc. Graw Hill. 2006. (captulo 11IIA 3 a (2), b (2), d (1) (8)). El sncope posprandial es causa comn de desvanecimiento en ancianos. El consumo de alcohol tambin puede producir sncope y a menudo es un factor contribuyente. Puede presentarse sncope posprandial cuando la sangre es desviada al lecho mesentrico, lo que produce disminucin relativa del riego cerebral. El paciente no muestra datos de actividad convulsiva; por tanto, es poco probable que se trate de una convulsin. Las arritmias cardacas y la hipersensibilidad del seno carotdeo pueden causar sncope y deben considerarse siempre en ancianos. Sin embargo, el medio en que este paciente se desvaneci indica que es ms probable un sncope posprandial. Un quiste coloide del tercer ventrculo es causa rara de prdida sbita de la conciencia. Los quistes pequeos actan como vlvulas de bola y obstruyen el flujo por el agujero de Monrow, lo que causa hidrocefalia aguda. 40. Una mujer de 36 aos es valorada porque presenta dolor de garganta y adenopata cervical. Su temperatura es de 37C, pulso de 90/min y presin arterial de 110/70 mmHg. El ganglio cervical anterior derecho mide 2.5 x 3 cm. No hay adenopata supraclavicular, axilar, epitroclear o inguinal palpable. La exploracin abdominal no es relevante. En los siguientes seis meses la paciente es revalorada por infecciones recurrentes de las vas respiratorias superiores. Durante este perodo hubo alguna regresin de ganglios cervicales, pero al parecer el tamao aumenta y disminuye en respuesta a los antibiticos. Por su adenopata persistente, la enferma es enviada con un cirujano y se realiza una biopsia del ganglio. Cul de los siguientes describe la histopatologa correspondiente al diagnstico de la enferma? (A) Linfoma folicular de clulas pequeas hendidas (B) Linfoma de clulas grandes difusas (C) Linfoma inmunoblstico (D) Linfoma de Burkitt

.- Allen R. M. MMS Medicina Interna. 5. Edicin. National Medical Series. Mc. Graw Hill. 2006. (captulo 4 XVI C 2, D 2). El linfoma folicular de clulas hendidas pequeas, llamado antes linfoma linfoctico nodular mal diferenciado, es una de las variedades ms comunes de linfomas no hodgkinianos indolentes. A menudo los pacientes presentan enfermedad en etapa III o IV. Aun sin tratamiento, los individuos pueden tener adenopata que aparece y desaparece, pero con el tiempo la enfermedad progresa y se requiere quimioterapia. Un pequeo porcentaje de sujetos presenta un linfoma ms agresivo, casi siempre de variedades de clulas grandes o inmunoblsticos. 41. En la infeccin por virus de la inmunodeficiencia humana por lo comn la linfadenopata difusa en una persona clnicamente sana suele ser un signo de: (A) Linfoma (B) Sarcoma de Kaposi (C) Tuberculosis (D) No indica infeccin o tumor especficos

20

Allen R. M. MMS Medicina Interna. 5. Edicin. National Medical Series. Mc. Graw Hill. 2006. (captulo 8 VIII G 1 b). La linfadenopata difusa en una persona infectada por virus de la inmunodeficiencia humana (VIH) que se encuentra clnicamente bien suele ser signo de que no hay infeccin especfica o tumor. Aunque todas esas respuestas pueden ser ciertas, las personas con mltiples ganglios aumentados de tamao, tuberculosis o trastornos malignos tienden a encontrarse enfermas. Con mayor frecuencia tambin experimentan prdida de peso y fiebre. Es ms probable que el linfoma se presente con afeccin orgnica en pacientes infectados con VIH que en otros. El sarcoma de Kaposi puede producir afeccin linftica, pero en general slo se encuentra en etapa tarda de la enfermedad con lesiones cutneas y mucosas extensas. La linfadenopata moderada es un dato comn en infeccin por VIH en etapa media. Se desconoce su causa exacta, pero la desaparicin de la linfadenopata prolongada puede preceder al deterioro clnico. La sfilis produce adenopata local o difusa en pacientes con infeccin por VIH o sin ella. Sin embargo, esta adenopata siempre se acompaa de algn otro dato de sfilis. 42. Cambios fisiopatolgicos durante el embarazo? (A) (B) (c) (D) Hiperuricemia Proteinuria Hipertensin Un incremento de 40% en la filtracin glomerular

Allen R. M. MMS Medicina Interna. 5. Edicin. National Medical Series. Mc. Graw Hill. 2006. (captulo 6, parte I: XV A 1, 2). El ndice de filtracin glomerular (GFR) se incrementa alrededor de 40% en el embarazo. De hecho hay un descenso de la concentracin srica de cido rico y un aumento de la depuracin de cido rico. No se ve proteinuria en el embarazo y el dato de una mayor excrecin urinaria de protenas sugiere la presencia de nefropata o preeclampsia subyacente. La presin arterial disminuye durante el embarazo; por tanto, cualquier grado de aumento representa hipertensin importante. Por ltimo, a menudo hay alcalosis respiratoria y no metablica y esto provoca una baja de la concentracin srica de bicarbonato. 43. Varn de 35 aos con poliuria se le practica una prueba de deshidratacin. Despus de la restriccin de lquidos, su osmolalidad urinaria mxima es de 550 mosm/kg y la plasmtica es de 295 mosm/kg. Luego de 1 h de aplicar la inyeccin subcutnea de 5 U de vasopresina acuosa, la osmolalidad urinaria es de 860 mosm/kg. Diagnstico probable de este paciente? (A) Sano (B) Diabetes inspida parcial (C) Diabetes mellitas (D) Diabetes inspida nefrgena Allen R. M. MMS Medicina Interna. 5. Edicin. National Medical Series. Mc. Graw Hill. 2006. (captulo 9 I B 1 c; cuadro 9-2). El paciente tiene diabetes inspida parcial. La respuesta definitiva del enfermo a la inyeccin de hormona antidiurtica (ADH) indica que no produce

21

concentraciones mximas eficaces de ADH despus de la restriccin de lquidos, y por tanto tiene diabetes inspida parcial o completa. La capacidad para lograr una concentracin urinaria normal o casi normal indica que el dficit de ADH slo es parcial. La respuesta de ADH descarta diabetes inspida nefrgena. La diabetes mellitus, otra causa de poliuria, se diagnostica por las concentraciones de glucosa en orina y sangre ms que por estudios del procesamiento renal de agua. 44. Es una de las enfermedades tiroideas autoinmunes organo-especfica ms frecuentes. Es ms comn en la mujer, posee una asociacin directa con otras enfermedades autoinmunes y, por lo general, se presenta con hipertiroidismo, bocio difuso, oftalmopata, adems, en algunos casos coincide con mixedema pretibial: (A) (B) (C) (D) Carcinoma basocelular Enfermedad de Graves Adenoma txico Sndrome del eutiroideo enferm

Halabe J. Mercado M. Nellen H. Tiroides. Gua prctica para el clnico Manual Moderno. Mxico. Primera Edicin. Pag. 52 53. La enfermedad de Graves es una de las enfermedades tiroideas autoinmunes rgano-especficas ms frecuentes. Se presenta con hipertiroidismo, bocio difuso, oftalmopata, adems, en algunos casos coincide con dermopata infiltrativa (mixedema pretibial), y acropaquia. Es tambin conocida como enfermedad de Parry o de Basedow. Es ms comn en la mujer, con una relacin mujeres hombres de 7 10 a 1. Adems de la predisposicin familiar para la enfermedad de Graves, hay una asociacin clnica w inmunolgica con otras enfermedades autoinmunes rgano-especficas: tiroiditis de Hashimoto, diabetes mellitus tipo 1, anemia perniciosa, vitiligo, falla ovrica primaria, insuficiencia adrenal y otras. 45. Javier de 49 aos de edad quien se refiere a s mismo como sano y sin molestias. Acude a una revisin general por indicacin de la empresa en la que labora. En sus estudios de laboratorio se encontr un aumento moderado de TSH (15mU/ml) con T3 y T4 normales. La primera posibilidad diagnstica es: (A) (B) (C) (D) Enfermedad de Graves Hipertiroidismo Hipotiroidismo subclnico Mixedema

.- Halabe J. Mercado M. Nellen H. Tiroides. Gua prctica para el clnico Manual Moderno. Mxico. Primera Edicin. Pag. 42. Mencin aparte merece el caso del aumento leve a moderado de TSH (5 a 20 mU/mL), con hormonas tiroideas normales, lo que generalmente ocurre en un paciente asintomtico y se conoce como hipotiroidismo subclnico. Sin embargo, es posible encontrar en estos pacientes fatiga, piel seca, sntomas neuromusculares o intolerancia al fro, con mayor frecuencia que en personas sanas del mismo sexo y edad. En ocasiones hay bocio o incluso oftalmopata distiroidea.

22

46. En nuestro pas consideramos en los enfermos con diabetes mellitus tipo 2 criterio de buen control metablico el siguiente: (A) Glucemia en ayuno < de 110 (mg/dl), glucemia postprandial de 2 hrs. < 140 (mg/dl), colesterol total < 200 (mg/dl), triglicridos en ayuno < 150 (mg/dl) y colesterol HDL > 40 (mg/dl) (B) Glucemia en ayuno < de 110 (mg/dl), glucemia postprandial de 2 hrs. < 200 (mg/dl), colesterol total < 200 (mg/dl), triglicridos en ayuno < 150 (mg/dl) y colesterol HDL > 40 (mg/dl) (C) Glucemia en ayuno > de 110 (mg/dl), glucemia postprandial de 2 hrs. > 200 (mg/dl), colesterol total < 200 (mg/dl), triglicridos en ayuno > 150 (mg/dl) y colesterol HDL > 40 (mg/dl) (D) Glucemia en ayuno < de 90 (mg/dl), glucemia postprandial de 2 hrs. < 200 (mg/dl), colesterol total < 200 (mg/dl), triglicridos en ayuno < 150 (mg/dl) y colesterol HDL > 40 (mg/dl)

MODIFICACION a la Norma Oficial Mexicana NOM-015-SSA2-1994, Para la prevencin, tratamiento y control de la diabetes mellitus en la atencin primaria para quedar como Norma Oficial Mexicana NOM-015-SSA2-1994, Para la prevencin, tratamiento y control de la diabetes.

APENDICE NORMATIVO E METAS BASICAS DEL TRATAMIENTO Y CRITERIOS PARA EVALUAR EL GRADO DE CONTROL DEL PACIENTE Metas del tratamiento Glucemia en ayunas (mg/dl) Glucemia postprandial de 2 h. (mg/dl) Colesterol total (mg/dl) Triglicridos en ayuno (mg/dl) Colesterol HDL (mg/dl) P.A. (mm de Hg) IMC HbA1c* Bueno <110 <140 <200.0 <150 >40 <120/80 <25 <6.5%mg/dl Regular 110-140 <200 200-239 150-200 35-40 121-129/81-84 25-27 6.5-8%mg/dl Malo >140 >240 >240 >200 <35 >130/85** >27 >8%mg/dl

23

* En los casos en que sea posible efectuar esta prueba. ** Es necesario un control estricto de la P.A. para reducir el riesgo de dao renal. Si el paciente fuma una meta adicional es dejar de fumar. 47. En un paciente diabtico no obeso en el cual existe fracaso en su control metablico posterior a la aplicacin de una dieta adecuada y un programa de ejercicio y que manifiesta francas polifagia, polidipsia y poliuria, la accin teraputica inmediata es? (A) (B) (C) (D) Iniciar de inmediato una sulfonilurea en dosis bajas Ingresar al hospital para iniciar manejo con insulina rpida subcutnea Iniciar con metformina e insulina rapida Si no se prefiere la terapia con insulina, aadir una biguanida

MODIFICACION a la Norma Oficial Mexicana NOM-015-SSA2-1994, Para la prevencin, tratamiento y control de la diabetes mellitus en la atencin primaria para quedar como Norma Oficial Mexicana NOM-015-SSA2-1994, Para la prevencin, tratamiento y control de la diabetes. APENDICE NORMATIVO I ESQUEMA DE TRATAMIENTO PARA EL PACIENTE DIABETICO NO OBESO

24

48 El mecanismo de accin de la glibenclamida: (A) (B) (C) (D) Estimula la liberacin de insulina de las clulas beta del pncreas Bloque la accin del pptido C con lo que disminuye la accin perifrica de la insulina Bloquea a la leptina Promueve la salida de GLUT 3

Goodman & Gilmans. The Pharmacology Diseases and Therapeutics. USA: Medical Publishing Division. Mc. Graw-Hill.2005: 1700-1707.2 . La Glibenclamida es un agente antidiabtico debido a su accin hipoglicemiante. Estimula la liberacin de insulina de las clulas del pncreas; aumenta los niveles de insulina mediante la reduccin de la liberacin heptica de la hormona. Incrementa la sensibilidad de los tejidos perifricos a la accin de la insulina y disminuye la glucogenlisis heptica y la gluconegnesis. Su efecto global es una reduccin de la concentracin sangunea de glucosa en pacientes diabticos cuyo pncreas es capaz de sintetizar insulina. 49. Son elementos fundamentales del diseo metodolgico en un ensayo clnico controlado:

25

(A) La asignacin es aleatoria, requiere la intervencin directa del investigador, necesariamente comparativo con control estricto de las variables, utiliza la maniobra ciega y proviene de una hiptesis matemtica (B) Retrospectivos, de observacin y comparativos. Parten del efecto a la causa y son dbiles por los sesgos de seleccin (C) Bloques de pacientes evaluados desde antes de presentar el efecto, requieren de una maniobra doble ciego, solo pueden medirse con estadstica descriptiva y poseen un alto valor en el estudio de enfermedades muy frecuentes (D) Solo utilizan una serie de casos

Crdova VH, Jimnez J, Jimnez MC. Manual de diseo metodolgico en investigacin clnica. ULSA UAPY 2001. Pag 30 31. El ensayo clnico controlado es un diseo muy cercano a un experimento, solo que en seres humanos. Busca respuestas concretas a cuestionamientos bien delimitados, utilizando intervenciones directas sobre la poblacin estudiada, relacionados con la curacin o el control de una medida teraputica, para lograrlo utiliza algunos elementos fundamentales, como: la asignacin aleatoria, maniobras doble ciego, la intervencin por parte del investigador, la comparacin entre los grupos, procedimientos de seguimiento muy rigurosos, as como, el consentimiento informado dentro de mrgenes biotico. 50. Los anti inflamatorios no esteroideos son frmacos muy utilizados, se absorben fcilmente en el tracto gastrointestinal, una consecuencia idiosincrsica importante que puede ocurrir an en pacientes sanos, es reversible si se detecta a tiempo y que debe considerarse con ms frecuencia en el caso del ibuprofeno es: (A) Elevacin de AST y ALT. (B) Cefalea. (C) Hipoglucmia (D) Nefrtis intersticial aguda.

Ruiz A, lvaro-Gracia J. Ma. Manual S.E.R. de las enfermedades reumticas. Sociedad Espaola de Reumatologa. Editorial Panamericana 2006. Pags. 183 188. Los anti inflamatorios no esteroideos poseen vida media y potencia distinta, esas caractersticas son expresiones de cada una de las formas moleculares, aquellos de vida media corta actan menos de seis horas, los de mayor tiempo de accin superan esta cifra. Entre las reacciones adversas ms frecuentes se encuentran las gastrointestinales, seguidas por aquellas en hgado, rin, piel y sistema nervioso central. La nefritis intersticial aguda acompaada de proteinuria masiva corresponde a una reaccin idiosincrsica que es ms frecuente con el uso de fenoprofeno. 51. La rigidez de las articulaciones interfalngicas proximales, meteacarpo falngicas, muecas, codos, rodillas, tobillos y metatarsofalngicas, una hora como mnimo, as como inflamacin simtrica de tres o ms de ellas, con imgenes con erosiones en manos al revisar imgenes simples de radiologa en posicin posteroanterior, son criterios de:

26

(A) Artertis esclerosante. (B) Artritis reactiva. (C) Artritis infecciosa. (D) Osteoartrosis. Ruiz A, lvaro-Gracia J. Ma. Manual S.E.R. de las enfermedades reumticas. Sociedad Espaola de Reumatologa. Editorial Panamericana 2006. Pags. 264 265. La rigidez de las articulaciones interfalngicas proximales, meteacarpo falngicas, muecas, codos, rodillas, tobillos y metatarsofalngicas por un periodo mnimo de una hora, as como la inflamacin simtrica de tres o ms de ellas, con imgenes con erosiones en manos y muecas al revisar imgenes simples de radiologa en posicin posteroanterior, as como, ndulos en prominencias seas, superficies extenoras o paraarticulares, con un factor reumatoide srico elevado, determinado por un mtodo que sea positivo en menos del 5% de los sujetos sanos normales son criterios diagnsticos de artritis reumatoide.

52.- Ubicacin frecuente del embarazo ectpico: (A) stmico (B) Ampular (C) Fmbrico (D) Intersticial

DeCherney A. (1999) Diagnstico y tratamiento ginecoobsttricos. Mxico. Ed. Manual Moderno. Pag 384. Los embarazos tubarios constituyen ms del 99% de los embarazos ectpicos: Las ubicaciones en las trompas son las siguientes: stmico (25%), ampular (55%), fmbrico (17%), intersticial (2%). El embarazo ovrico ocurre en <0.5 y los abdominales <0.1%.

53.- Fem. 35 aos G3 C3 presenta manchado durante el primero y segundo trimestres y a las 33 SDG hemorragia abundante, repentina e indolora, su principal sospecha es? (A) Placenta previa (B) Ruptura uterina (C) Coriocarcinoma (D) Desprendimiento grave de placenta normoinserta

DeCherney A. (1999) Diagnstico y tratamiento ginecoobsttricos.Mxico. Ed. Manual Moderno. Pag 501 Las bases para el diagnstico de placenta previa son: manchado durante el primero y segundo trimestres, hemorragia abundante, repentina e indolora en el tercer trimestre, clicos iniciales

27

en 10% de las pacientes. La frecuencia de placenta previa aumenta con la edad, multiparidad y cesreas previas.

54. Esteroide de origen principalmente placentario: (A) Progesterona (B) Estrona (C) Estradiol (D) ACTH

DeCherney A. (1999) Diagnstico y tratamiento ginecoobsttricos.Mxico. Ed. Manual Moderno. Pag 187. La progesterona es el esteroide de origen principalmente placentario, la estrona y el estradiol son hormonas de origen maternoplacentofetal, la lipotropina y la ACTH son de origen fetales.

55. Acude a su consulta paciente embarazada de 28 aos G1, a las 30 SDG, refiere ardor y prurito vaginal intensos que se agravan con el coito. A la exploracin se observa secrecin blanca caseosa profusa e inflamacin de la vagina y el introito, Su sospecha diagnstica es? (A) Clamidiasis (B) Candidiasis (C) Tricomoniasis (D) Sfilis

DeCherney A. (1999) Diagnstico y tratamiento ginecoobsttricos .Mxico. Ed. Manual Moderno. Pag 237. Candida albicans puede cultivarse desde la vagina en muchas embarazadas los sntomas se presentan en menos de 50%. Cuando los hay consisten en ardor y prurito vaginal intensos y secrecin blanca caseosa profusa. Quiz se encuentre inflamacin notable de vagina e introito. Es posible que el coito agrave los sntomas.

56. Acude a su consulta femenina de 26 aos que cursa el tercer trimestre del embarazo tiene edema en miembros inferiores sin sintomatologa agregada. Su primer medida teraputica en esta paciente es? (A) Elevacin de los miembros inferiores en decbito lateral (B) Restriccin Hdrica (C) Ti cidas

28

(D) Diurtico de asa

DeCherney A. (1999) Diagnstico y tratamiento ginecoobsttricos. Mxico. Ed. Manual Moderno. Pag 237. El edema de las partes bajas producido por la impedancia del retorno venoso es comn en la parte final del embarazo, La paciente debe tratarse solo si est molesta. La elevacin de los miembros inferiores (especialmente en decbito lateral) mejora la circulacin. Estn contraindicados los diurticos porque pueden ser peligrosos.

57. Agente etiolgico del chancro blando: (A) Haemophilus ducreyi (B) Calymmatobacterium granulomatosis (C) Neisseria gonorrhoeae (D) Chlamydia trachomatis

Scout J. (1997) Danforth. Manual de Obstetricia y Ginecologa. Mxico. Ed. Mc Grqw Hill Interamericana. Pag 388 El agente etiolgico del chancro blando es Haemophilus ducreyi.

58. Si en la etapa prepuberal persiste la vulvovaginitis inespecfica a pesar de tratamiento, es necesario descartar: (A) Cuerpo extrao (B) Traumatismo (C) Pubertad precoz (D) Tumor de clulas germinales

Berek J. (2002) Ginecologa de NOVAK. Mxico. Ed. Mc Graw Hill Interamericana. Pag 291 Si se cree que la secrecin sanguinolenta es causada por vulvovaginitis inespecfica y persiste a pesar del tratamiento quiz se requiera valoracin ms a fondo para descartar la presencia de cuerpo extrao.

59. Ante hemorragia irregular o postcoital en una mujer adolescente usted debe sospechar cervicitis por:

29

(A) Chlamydia (B) Micoplasma (C) Gardnerella (D) Candida

Berek J. (2002) Ginecologa de NOVAK. Mxico. Ed. Mc Graw Hill Interamericana. Pag 293. La hemorragia irregular o poscoital puede relacionarse con cervicitis por Chlamydia. La adolescentes tienen las tasas ms altas de infecciones por Chlamydia que cualquier grupo de edad, y deben investigarse estos microorganismos de manera sistemtica entre las adolescentes sexualmente activas.

60. Se refiere a una menstruacin anormal de intervalo regular, duracin prolongada y cantidad excesiva: (A) Menorragia (B) Metrorragia (C) Menometrorragia (D) Hipermenorrea

Berek J. (2002) Ginecologa de NOVAK. Mxico. Ed. Mc Graw Hill Interamericana. Pag 298. En la menorragia el intervalo es regular, tiene duracin prolongada y cantidad excesiva.

61. Se asocia frecuentemente a endometriosis: (A) Dismenorrea (B) Dispareunia severa y dolor plvico crnico (C) Infertilidad (D) Embarazo ectpico

Beck W. (1997) NMS. Obstetrics and Gynecology. USA. Williams & Wilkins. Pag 277. La endometriosis se asocia frecuentemente con: dismenorrea, dispareunia severa, dolor plvico crnico e infertilidad.

62. Sus niveles estn elevados en aquellos pacientes con endometriosis:

30

(A) CA-125 (B) Tc 99 (C) AFP (D) TGP

Beck W. (1997) NMS. Obstetrics and Gynecology. USA. Williams & Wilkins. Pag 278. Se encuentra elevacin srica de CA-125 en pacientes con endometriosis, se trata de un antgeno de superficie encontrado en derivados del epitelio celmico, incluyendo el endometrio, se correlacionan sus niveles con el grado de enfermedad y la respuesta a tratamiento, pudiendo ser un marcador de recurrencia de la enfermedad.

63. Es un factor que incrementa infecciones de vas urinarias: (A) Preservativos (B) Espermicidas (C) Esponjas vaginales (D) Diafragmas

Beck W. (1997) NMS. Obstetrics and Gynecology. USA. Williams & Wilkins. Pag 243. Existen pocos efectos secundarios al uso del diafragma. Se reporta un incremento en la frecuencia de infecciones de vas urinarias probablemente resultado de compresin uretral.

64. El absceso tubo-ovrico unilateral se asocia con: (A) Preservativos (B) Esponjas vaginales (C) Diafragmas (D) DIU

Beck W. (1997) NMS. Obstetrics and Gynecology. USA. Williams & Wilkins. Pag 245. El absceso tubo-ovrico unilateral se encuentra ocasionalmente en pacientes con DIU.

65. El tromboembolismo como complicacin de la administracin de estrgenos se debe a: (A) Incremento plasmtico del factor VII principalmente (B) Trombocitosis (C) Plaquetopenia (D) Neutropenia

31

Beck W. (1997) NMS. Obstetrics and Gynecology. USA. Williams & Wilkins. Pag 245-246. El estrgeno causa un incremento en los niveles plasmticos de factores de coagulacin, especialmente el factor VII, probablemente actuando a nivel heptico. Los niveles de antitrombina III descienden en los 10 das de inicio de los anticonceptivos orales. La incidencia de trombosis venosa superficial y profunda se encuentra incrementada en consumidoras de anticonceptivos orales.

66. Con respecto a los anticonceptivos orales (estrgeno/progestinas): (A) Las progestinas son agonistas de los estrgenosen su efecto estimulante sobre el tejido mamario (B) Se observa un aumento en la incidencia de enfermedad mamaria benigna con su uso (C) Las progestinas compiten con el estrgeno por los sitios de unin en las clulas endometriales (D) Los estrgenos disminuyen el efecto proliferativo en el endometrio de las progestinas

Beck W. (1997) NMS. Obstetrics and Gynecology. USA. Williams & Wilkins. Pag 247. Las progestinas compiten con el estrgeno por los sitios de unin en las clulas endometriales. Las progestinas antagonizan el efecto estimulante del estrgeno sobre el tejido mamario. Hay una disminucin en la incidencia de enfermedad mamaria benigna en las consumidoras de anticonceptivos orales. Las progestinas reducen el efecto estimulante del estrgeno y previenen la proliferacin normal del endometrio prolifetativo hacia hiperplasia. Los quistes funcionales son menos en las consumidoras de anticonceptivos orales.

67. Causal para suspender los anticonceptivos con solo progestina? (A) Alteraciones menstruales (B) Eversin cervical (C) Hiperplasia endometrial (D) Incremento de peso

Beck W. (1997) NMS. Obstetrics and Gynecology. USA. Williams & Wilkins. Pag 247. La principal razn por la que se interrumpe el consumo de anticonceptivos con solo progestina es por alteraciones menstruales.

68. De los siguientes cual suprime la ovulacin como mecanismo de accin?

32

(A) DIU (B) Norgestrel y etinil estradiol postcoital (C) Levonorgestrel (D) Agonistas de GnRH

Beck W. (1997) NMS. Obstetrics and Gynecology. USA. Williams & Wilkins. Pag 249. Los agonistas de GnRH de forma primaria suprimen la ovulacin. El DIU con progesterona evita que el endometrio mantenga la implantacin y favorece la formacin de moco cervical de caractersticas hostiles para la migracin espermtica. El norgestrel y el etinil estradiol postcoital causan disrupcin en el endometrio previniendo la implantacin. El levonorgestrel puede suprimir la ovulacin pero tambin ejerce tiene efectos sobre el endometrio en la misma forma que el DIU. La mifepristona se une al receptor de la progesterona e interrumpe la accin progestacional a travs de lisis del cuerpo lteo.

69. Pueden lograr una disminucin del flujo menstrual: (A) Ibuprofeno (B) Acido mefenmico (C) Estrgenos y progestgenos. (D) Acido acetil saliclico 69. La respuesta es C.- Berek J. (2002) Ginecologa de NOVAK. Mxico. Ed. Mc Graw Hill Interamericana. Pag 302. El ibuprofeno y el cido mefenmico disminuyen el flujo menstrual. La hemorragia anormal puede controlarse a menudo con tratamiento hormonal cuando es excesiva o irregular. En los pacientes en quienes estn contraindicados los estrgenos, se pueden usar pogestgenos para controlar la hemorragia excesiva. 70. Indicacin para la administracin de rgimen con estrgenos: progestgenos en los ltimos 10 a 13 das de un

(A) Disminuir el riesgo de hiperplasia y neoplasia del endometrio (B) Disminuir el incremento de peso (C) Disminuir la mastalgia (D) Disminuir el riesgo de neoplasia mamaria maligna

Berek J. (2002) Ginecologa de NOVAK. Mxico. Ed. Mc Graw Hill Interamericana. Pag 303 Las mujeres que estn recibiendo tratamiento de restitucin hormonal durante la menopausia pueden recibir estrgenos los primeros 25 das de cada mes. Se aade un progestgeno a menudo acetato de medroxiprogesterona, durante los 10 a 13 ltimos das de este rgimen en un esfuerzo por reducir el riesgo de hiperplasia y neoplasia del endometrio.

33

71. Alteracin en la enfermedad de Addison? (A) Hipernatremia (B) Hiperkalemia (C) BUN bajo (D) Orina diluda

. McPhee S, Papadakis M, Tierney L. currrent Medical diagnosis and treatment. McGraw Hill 2007 1189. 1190. La enfermedad de Addison se produce por disfuncin o destruccin de las glndulas suprarrenales, se caracteriza por deficiencia de cortisol, aldosterona y andrgenos. Se observa hiponatremia, deplecin de volumen e hiperkalemia. Puede presentarse Hipercalcemia e hipoglucemia, as como moderada neutropenia, linfocitosis y una cuenta de eosinfilos de 300/mcl. Los sntomas incluyen debilidad, fatiga, prdida de peso, mialgias, artralgias, fiebre, anorexia, nusea, vmito, hipotensin, ansiedad e irritabilidad mental. As como hiperpigmentacin de cuello, codos, rodillas, palmas, pezones y las lneas de presin.

72. Que porcentaje del peso corporal corresponde al espacio intracelular? (A) 20 al 10% (B) 70 al 80% (C) 30 a 40% (D) 20 a 25%

Bruce E. Jarrell, R. Anthony Carabasi, Nacional Medical Series for Independent Study. Wiliams & Wilkins, 3rd Edition: 1-25. El compartimento intracelular forma del 30 al 40% del peso corporal total (65% del agua corporal total). La mayora del agua intracelular se encuentra alojada en la clula msculoesqueltica, un porcentaje muy pequeo en el adiposito.

34

73. causa de hiponatremia intra-hospitalaria es de tipo iatrgeno, y sus sntomas son: (A) Irritabilidad, temblor muscular, crisis convulsivas y disminucin de los reflejos tendinosos. (B) Irritabilidad, temblor muscular, crisis convulsivas y aumento de los reflejos tendinosos. (C) Irritabilidad, debilidad muscular, crisis convulsivas y aumento de los reflejos tendinosos. (D) Astenia, temblor muscular, crisis convulsivas y aumento de los reflejos tendinosos.

Bruce E. Jarrell, R. Anthony Carabasi, N acional Medical Series for Independent Study. Wiliams & Wilkins, 3rd Edition: 1-25. Los sntomas de la hiponatremia son generalmente, irritabilidad, aumento de los reflejos tendinosos, temblor muscular, crisis convulsivas. La hiponatremia se define como niveles de sodio por debajo de 130 mEq/L.

74. Principal causa de hipocalcemia? (A) Iatrgena (B) Exceso de lquidos parenterales (C) Hiperparatiroidismo (D) Ciruga de paratiroides.

Bruce E. Jarrell, R. Anthony Carabasi, N acional Medical Series for Independent Study. Wiliams & Wilkins, 3rd Edition: 1-25. La causa ms comn de hipocalcemia es la reseccin quirrgica de gandulas paratiroides secundario a hiperparatiroidismo. Algunas otras causas intrahospitalarias son malnutricin crnica, administracin de soluciones cristaloides a grandes volmenes y transfusiones de productos hemticos.

75. A que nos referimos con la fase ebb de la respuesta metablica al trauma? (A) Es la fase final de la respuesta (B) No existe esa fase (C) Es la fase media de la respuesta (D) Es la fase inicial de la respuesta

Schwartz, Principles of Surgery. 7th Edition, McGraw Hill 1999: 3-51 La fase ebb corresponde al inicio de la respuesta metablica al trauma, sucede en las primeras horas posteriores a la lesin y generalmente se asocia a inestabilidad hemodinmica o a una

35

reduccin en el volumen efectivo circulante. En esta fase existe una reduccin importante en el gasto energtico total y en las prdidas de nitrogeno urinario. Esta fase se caracteriza por una elevacin de catecolaminas y cortisol.

76. En una fstula gstrica el principal electrolito que se pierde es? (A) Na (B) K (C) Cl (D) Ca

Humberto Arenas Marques, Roberto Anaya Prado. Nutricin enteral y parenteral. McGraw Hill 2007:13-22 La prdida de electrolitos a nivel gstrico es de 60 mEq de sodio, 10 mEq de Potasio, 90 mEq de cloro y 0 mEq de Hco3 77. En una fstula Pancretica el principal electrolito que se pierde es? (A) Na (B) K (C) Cl (D) Ca y Mg

Humberto Arenas Marques, Roberto Anaya Prado. Nutricin enteral y parenteral. McGraw Hill 2007:13-22 La prdida de electrolitos a nivel pancretico es de 140 mEq de sodio, 5 mEq de Potasio, 75 mEq de cloro y 90 mEq de Hco3.

78. El electrolito ms abundante en la saliva es? (A) Na (B) K (C) Cl (D) Ca

Humberto Arenas Marques, Roberto Anaya Prado. Nutricin enteral y parenteral. McGraw Hill 2007:13-22 La prdida de electrolitos en saliva es de 30 mEq de sodio, 20 mEq de Potasio, 95 mEq de cloro y 15 mEq de Hco3.

36

79. Indicacin para iniciar nutricin parenteral? (A) Pancreatitis (B) Colitis Ulcerativa Crnica Inespecfica (CUCI) (C) Reseccin intestinal posquirrgica (D) Oclusin intestin

Humberto Arenas Marques, Roberto Anaya Prado. Nutricin enteral y parenteral. McGraw Hill 2007:243-250 La nutricin parenteral total esta indicada en pacientes con presencia de enfermedades tales como pancreatitis, enfermedades inflamatorias intestinales, reseccines intestinales posquirrgicas que no tienen posibilidad de alimentacin distal a la anastomosis, y cncer entre otras; pero una de las indicaciones absolutas para la aplicacin de nutricin parenteral total es la oclusin intestinal, debido a la imposibilidad de utilizar el tubo digestivo.

80. Caracterstica de las heridas limpias-contaminadas? (A) Sucede cuando previo a la herida existe evidencia de infeccin (B) Es la herida que se genera dentro de un quirfano (C) Es una herida limpia que se realiza en quirfano pero que involucra tejidos gastrointestinal, respiratorio y/o genitourinario (D) Es una herida limpia realizada en quirfano que posteriormente se contamina

como

Bruce E. Jarrell, R. Anthony Carabasi, Nacional Medical Series for Independent Study. Wiliams & Wilkins, 3rd Edition: 1-25. La clasificacin actual de las heridas es: Limpias, Limpias-contaminadas, contaminadas e infectadas. Las heridas limpias son aquellas que se realizan bajo tcnica estril dentro de quirfano. Las heridas limpias-contaminadas son aquellas heridas limpias que involucran tejidos potencialmente contaminantes como tubo digestivo, genitourinario y/o pulmonar. Las contaminadas son aquellas donde hay evidencia de contaminacin mayor por presentarse durante una infeccin severa del rgano involucrado (anastomosis intestinal en peritonitis severas). La herida infectada es aquella que se presenta la infeccin previo a la creacin de la herida como en abscesos apendiculares.

81. Formula para el calculo de requerimientos calricos postquirrgicos? (A) 20 Kcal/kg/da (B) 50 Kcal/kg/da (C) 30 Kcal/kg/da

37

(D) 60 Kcal/kg/da

Bruce E. Jarrell, R. Anthony Carabasi, Nacional Medical Series for Independent Study. Wiliams & Wilkins, 3rd Edition: 1-25. El gasto calrico total calculado para cualquier enfermo de inicio y si no contamos con calorimetra se debe realizar con la frmula de 30 kcal/kg/da. Este gasto durante el ayuno disminuye al mximo para mantener funciones vitales, pero durante el estrs, se puede elevar a ms del doble.

82. Complicacin pulmonar postquirrgica? (A) Neumona (B) Atelectasias (C) Derrame pleurales (D) Neumotrax

Bruce E. Jarrell, R. Anthony Carabasi, Nacional Medical Series for Independent Study. Wiliams & Wilkins, 3rd Edition: 27-50. Las atelectasias por mucho son la complicacin ms frecuente de los pacientes posoperados la cual se trata con medidas generales como inspirometra incentiva y fisioterapia pulmonar. La complicacin pulmonar que le contina en frecuencia son los procesos neumnicos.

83. Contraindicacin para iniciar alimentacin enteral ? (A) Pancreatitis (B) Anastomosis intestina (C) Isquemia intestinales (D) Insuficiencia cardiaca congestiva

Humberto Arenas Marques, Roberto Anaya Prado. Nutricin enteral y parenteral. McGraw Hill 2007:261-270 La isquemia intestinal es una contraindicacin absoluta para la administracin de nutricin enteral debido a que la distensin y la presencia de nutrientes favorecer la presencia de isquemia intestinal y mayor dao.

38

84. Recin nacido de trmino en su segundo da de vida, que presenta una dermatosis diseminada a cara y tronco caracterizada por vesculas y pstulas, que respeta palmas y plantas. Su sospecha diagnostica es? (A) Melanosis pustulosa (B) Eritema txico (C) Dermatitis atpica (D) Rubola congnita

Manual CTO Pediatra, 7 edicin, p. 1295 El eritema txico es una entidad sin significado patolgico que se manifiesta con vesculas y pstulas sobre una base eritematosa, respeta palmas y plantas. Aparece entre el primero y tercer da de vida. Estn formadas por un infiltrado de eosinfilos, el cultivo es estril y desaparece en la primera semana. En cambio, la melanosis pustulosa se presenta al nacimiento, se localiza en palmas y plantas, las lesiones estn formadas por PMN, el cultivo es estril y desaparecen en varias semanas.

85. Factores de riesgo en el recin nacido para hemorragia suprarrenal: (A) Macrosmicos y nacidos por parto de nalgas. (B) Macrosmicos y nacidos por cesrea. (C) Prematuro y nacidos por parto de nalgas. (D) Prematuro y nacidos por cesrea.

Manual CTO Pediatra, p.1295. La hemorragia suprarrenal afecta a productos macrosmicos nacidos por parto de nalgas y se manifiesta con anemia, hipoglucemia, trastornos electrolticos y anemia.

86. Paciente masculino de 10 aos de edad que inici su padecimiento hace 24 hrs al presentar, posterior a la ingesta de pescado, evacuaciones diarreicas, dolor abdominal, nusea y vmito; acompandose de cefalea, parestesias periorales y bradicardia. El da de hoy se agreg la presencia de ataxia y refiere que se quema al tocar el agua fra. Cul es el agente etiolgico ms probable? (A) Vibrio cholerae (B) Agente Norwalk (C) Ciguatera (D) StaphIlococcus aeurus

39

U.S. Food & Drug Administration - Center for Food Safety & Applied Nutrition - Foodborne Pathogenic Microorganisms and Natural Toxins Handbook. La intoxicacin por ciguatera se produce por ingerir pescados como dorado, barracuda, cubera, etc. contaminados con macroalgas con ciguatoxina-1. la evolucin de la intoxicacin es la siguiente: en las primeras 12 horas se presentan manifestaciones gastrointestinales como dolor abdominal, nusea, vmito y diarrea; posteriormente se agregan datos neurolgicos como calambres, parestesias, mialgias, fatiga, ataxia, y disgeusia y finalmente alteraciones cardiovasculares (bradicardia, taquicardia o hipotensin.) Los datos principales son las parestesias periorales y la inversin de la sensacin trmica, es decir sentir fro al tocar objetos calientes y viceversa. Adems se puede presentar ceguera temporal, parlisis de los msculos faciales, oftalmoplejia, espasticidad, delirio, sensacin de prdida de los dientes, exacerbacin del acn, sialorrea, cada del cabello, uas y descamacin de las piel.

87. Recin nacido de 32 semanas de gestacin, que presenta de manera sbita hipotensin, anemia y abombamiento fontanelar. A al EF presenta una FC de 100 x, una FR de 20 x y comienza a convulsionar El diagnstico probable es? (A) Hemorragia de La matriz germinal (B) Leucomalacia periventricular (C) Hidrocefalia (D) Hipotiroidismo congnito

.- Manual CTO pediatra, 7 edicin p.1295 La matriz germinal es una estructura del cerebro inmaduro hasta la semana 34 de gestacin. Su lesin produce hemorragia. Se manifiesta con deterioro del estado general, aparicin de pausas de apnea, bradicardia, hipoventilacin, convulsiones, disminucin del reflejo del Moro y fontanelas a tensin. Debe sospecharse ante un paciente pretrmino que presenta de forma sbita anemia, hipotensin y abombamiento fontanelar. Puede derivar en la aparicin de leucomalacia periventricular.

88. Recin nacido de trmino que presenta reflejo de moro asimtrico a expensas de inmovilidad del brazo derecho; el resto de los reflejos no presentan alteraciones. La madre refiere que durante el parto present distocia de hombros. Cul es el diagnstico? (A) Parlisis de Djerine Klumpke. (B) Parlisis de Erb Duchenne (C) Hemiparesia derechas (D) Lesin de plexo braquial

40

Manual CTO pediatra, 7 edicin p.1299. La parlisis de Erb Duchenne se presenta en recin nacidos que nacen por parto con distocia de hombros, provocando una hiperextensin lateral del cuello, lesionando las races de C5 y C6; por lo que se manifiesta con adduccin y rotacin interna del brazo ipsilateral, el reflejo del Moro es asimtrico. En caso de afectarse la raz de C4 se agregar parlisis frnica. En cambio, la parlisis de Djerine - klumpke se presenta en las distocias de nalgas, con hiperextensin del brazo, se lesionan las races de C7 y C8, manifestndose con mano cada y ausencia del reflejo de prensin palmar del lado afectado; puede involucrar la raz de T1 originando sndrome de Horner.

89. Factores de riesgo para enterocolitis necrotizante: (A) Anemia, hipoxia, bajo gasto e infeccin por Yersinia enterocoltica (B) Policitemia, alto gasto e infeccin por Shigella (C) Policitemia, hipoxia, bajo gasto e infeccin por Escherichia coli (D) Prematurez, corioamnioitis, isoinmunizacin materno- fetal.

Manual CTO pediatra, 7 edicin p.1306. La enterocolitis necrotizante es una lesin isqumico necrtica que afecta al leon distal y colon proximal. Las causas que predisponen a la isquemia son: policitemia, inicio temprano de la alimentacin con elevados volmenes y concentraciones y situaciones de hipoxia y bajo gasto. Se han involucrado a los siguientes agentes: E. coli, Clostridium prefringens, Staphilococcus epidermidis y rotavirus. Se manifiesta con distensin abdominal y evacuaciones sanguinolentas en la segunda semana de vida, que puede originar sepsis, choque y muerte. En la radiografa de abdomen, el dato tpico es la neumatosis intestinal, tambin se presenta edema de asas, patrn en miga de pan, asa fija y gas en la vena porta. El tratamiento consiste en antibiticos para Gram negativos y anaerobios, ayuno, descompresin nasogstrica, aporte de lquidos y ciruga en caso necesario.

90. Recin nacido de 33 semanas de gestacin que presenta en el primer da de vida, taquipnea, retraccin xifoidea, tiraje intercostal, aleteo nasal, quejido intenso y cianosis. A la exploracin fsica se auscultan crepitantes bibasales. La placa de trax muestra infiltrados reticulonodulares con broncograma areo. Cul es el diagnstico probable? (A) Neumona (B) Enfermedad de membrana hialina (C) Taquipnea transitoria del recin nacido (D) Sndrome de aspiracin de meconio

41

Manual CTO pediatra, 7 edicin p. 1302. La enfermedad de membrana hialina se debe a un dficit de surfactante, afecta a recin nacidos prematuros, su frecuencia aumenta en hijos de madres diabticas y en embarazos mltiples. El cuadro clnico se caracteriza por datos de dificultad respiratoria de inicio precoz como0: taquipnea, quejido intenso, aleteo nasal, retracciones xifoideas e inter y subcostales y cianosis parcialmente refractaria al oxgeno. Los sntomas progresan hasta alcanzar un mximo al tercer da. En la auscultacin aparecen crepitantes en ambas bases. En la radiografa de trax se presenta un infiltrado reticulonodular con broncograma areo en unos pulmones poco ventilados, se pueden presentar atelectasias. En la gasometria se observa hipoxemia, hipercapnia y acidosis respiratoria. El tratamiento consiste en soporte respiratorio, administracin endotraqueal de surfactante y antibiticos. Por otro lado, la taquipnea transitoria delr ecin nacido se presenta en recin nacidos de trmino, que nacen por cesrea o parto vaginal rpido, debido a un retraso en la absoprcin del lquido; manifestndose con un distrs respiratorio de inicio precoz, la auscultacin es normal. 91. Caractersticas de la ictericia fisiolgica? (A) Inicia entre el segundo y tercer da de vida, tiene una duracin menor de 10 das y la concentracin mxima es de 12 mg/dl (B) Inicia el primer da de vida, tiene una duracin menor de 10 das y la concentracin mxima es de 20 mg/dl (C) Inicia entre el segundo y tercer da de vida, tiene una duracin mayor de 10 das y la concentracin mxima es de 12 mg/dl. (D) Inicia a la semana de nacimiento y dura una semana ms.

Manual CTO pediatra, 7 edicin p. 1306. La ictericia fisiolgica del recin nacido inicia entre el segundo y tercer da de vida, tienen una duracin entre 5 y 7 das y alcanza una concentracin mxima de 12 mgdl a los 2 4 das. En cambio la ictericia no fisiolgica inicia dentro de las primeras 24 horas de vida, tiene una duracin superior de 10 a 15 das, la bilirrubina total en RNT es mayor de 12 mg/dl o mayor de 14 mg/dl en RNPT, el incremento de bilirrubina es mayor de 5 mg/dl en 24 horas y la bilirrubina directa es mayor de 1 mg/dl.

92. Se le debe aplicar profilaxis con gammaglobulina humana anti-D. (A) Madre Rh negativo, padre Rh positivo, con prueba de Coombs indirecta negativa (B) Madre Rh negativo, padre Rh positivo, con prueba de Coombs indirecta positiva (C) Madre Rh negativo, padre Rh negativo, con prueba de Coombs indirecta positiva (D) Madre Rh positivo, padre Rh negativo, con prueba de Coombs indirecta positiva

Manual CTO pediatra, 7 edicin p 1307.

42

La enfermedad hemoltica por incompatibilidad Rh se produce cuando la madre es Rh negativo y el feto es Rh positivo. La madre se sensibiliza frente al antgeno D cuando hay paso de hemates fetales a la circulacin materna, produciendo anticuerpos IgM anti D que atraviesan la placenta y destruyen los hemates fetales el primer producto no se afecta, sino hasta el segundo en el que se producen anticuerpos IgG. Se manifiesta con ictericia, anemia hemoltica e hidrops fetal. La positividad del test de Coombs indirecto indica que la madre est sensibilizada frente al antgeno D; por lo que la profilaxis con IgM anti D se aplica a las mujeres con test de Coombs negativo a las semana 28 de gestacin y en las primeras 72 horas despus del parto, aborto o amniocentesis.

93.- En cul de las siguientes infecciones conntales se presenta la trada de Gregg? (A) Rubola (B) Toxoplasmosis (C) Citomegalovitus (D) Sfilis congnita Manual CTO pediatra, 7 edicin p 1312, 1313. La trada de Gregg que consiste en hipoacusia neurosensorial, malfiormaciones congnitas (PCA o estenosis pulmonar) y anomalas oculares (catarata, glaucoma, retinitis en sal y pimienta) se presenta en la rubola; otras manifestaciones son: corioretinitis, prpura trombocitopnica y microcefalia. En la infeccin por citomegalovirus tambin se presenta microcefalia y coriorretinitis, as como calcificaciones periventriculares.

94.- Es manifestacin de la sfilis congnita tarda: (A) Hepatomegalia (B) Rinorrea (C) Queratitis (D) Lesiones cutneas polimorfas

Manual CTO pediatra, 7 edicin p 1313. La sfilis congnita tarda se manifiesta despus de los 2 aos de vida, el sntoma ms frecuente es la queratitis, que es parte de la trada de Hutchinson (queratitis, hipocausia y dientes en tonel o Hutchinson), as como deformidades seas (tibias en sable, frente olmpica, engrosamiento clavicular), rodilla de clutton, paresia juvenil y tabes dorsal. La hepatomegalia, rinorrea, roseola sifiltica y la pseudoparlisis de parrot son manifestaciones de la sfilis precoz que se manifiesta en los primeros dos aos de vida.

43

95. La mejor manera de diagnosticar alteraciones prostticas es? (A) Ultrasonido trans-rectal (B) Toma de antgeno prosttico especifico (C) Tacto rectal (D) Biopsia prosttica

Bruce E. Jarrell, R. Anthony Carabasi, Nacional Medical Series for Independent Study. Wiliams & Wilkins, 3rd.La mejor manera para detectar alteraciones a nivel prosttico es a la palpacin directa con un tacto rectal, los otros estudios sirven para confirmar el diagnstico o sospecha y normar conducta a seguir.

96. En la litiasis renoureteral: Cules son los clculos radiolcidos? (A) De calcio (B) De estruvita (C) De cido rico (D) De cistina

Bruce E. Jarrell, R. Anthony Carabasi, Nacional Medical Series for Independent Study. Wiliams & Wilkins, 3rd Edition: 451-475. Los clculos que sepueden presentar en la va urinaria son formados por calcio, cido rico y cistina, los nicos radiolucidos son los de cido rico que representan un reto diagnstico.

97. Ante la sospecha de una torsin testicular, prueba diagnstica de eleccin? (A) Ecografa-doppler (B) Ecografa renal (C) Tomografa renales (D) Arteriografa renal selectiva

Smith, Urologa General 11 Ed., pgs. 94-97 La ecografa es el estudio de eleccin para el diagnstico definitivo de torsin testicular.

98. Lactante de 5 meses que es trado a consulta por presentar desde 3 das antes, fiebre, rinorrea acuosa y estornudos, comenzando el da de la consulta con tos y dificultad

44

respiratoria. En la exploracin presenta taquipnea, tiraje subcostal, alargamiento de la espiracin, sibilancias diseminadas. Prueba, que solicitara para determinar la etiologa? (A) Biometra Hematica (B) Radiografa de trax (C) Electrolitos en sudor (D) Bsqueda de virus respiratorio sincitial en exudado nasofarngeo

Nelson, Tratado de Pediatra 15 Ed., pgs. 1521-1522. Un cuadro infeccioso en un lactante (fiebre, rinorrea y estornudos), en el que hay localidad respiratoria (tos y dificultad respiratoria) y signos de dificultad en la espiracin, debe hacer pensar en el diagnstico de Bronquiolitis. Se presenta en lactantes entre 3 y 6 meses de edad. Suele responder a una etiologa viral, lo ms frecuente es el Virus Sincitial Respiratorio

99. En un nio sano de 3 meses, el valor de la hemoglobina es: (A) 3-17 g/dL (B) 9.5-14 g/dL (C) > 15 g/dL (D) < 9 g/dL

Nelson, Tratado de Pediatra 15 Ed., pgs. 325-326. Las cifras de Hemoglobina tienen una evolucin muy tpica durante la infancia. El recin nacido de trmino (RNT) tiene una cifras de hemoglobina de 14-20 g/dL, el RN de muy bajo peso tiene 1-2 g/dL menos. A partir de las 48h, comienza a descender la Hb. En el RNT hacia las 12 semanas (3 meses) de vida se produce un descenso fisiolgico de Hb, alcanzando cifras mnimas de 9-11g/dL y hacia las 6 semanas en el recin nacido pretrmino con cifras de 7-10 g/dL. A partir de esa edad, las cifras de Hb empiezan a ascender paulatinamente, estabilizndose en la edad adulta. Ante niveles menores a lo esperado de Hb, hablamos de anemia. La anemia en los primeros das generalmente es causada por hemlisis de los glbulos rojos y dficit de eritropoyetina, mientras que en el resto de la infancia suele ser ferropnica.

100. Un mdico, que no dispone de tablas de crecimiento y desarrollo, necesita valorar el peso de un nio de 3 aos. Qu frmula entre las siguientes, debera utilizar para calcular el peso medio de un nio normal de esa edad? (A) Edad en aos x 8 / 2 (B) Edad en aos x 80 / 2 (C) (Edad en aos x 2) + 8 (D)Edad en meses x 10 / 2

45

Nelson, Tratado de Pediatra 15 Ed, vol 1., pg. 35. Un nio sano duplica su peso a los 5 meses, lo triplica a los 12 meses y cuadriplica su peso a los 2 aos (pesando 12 a 14kg). La respuesta C es la ms cercana al valor esperable, ya que la talla al nacimiento es de 50cm, al los 12 meses: 50 + 50/2 = 75 cm; a los 24 meses: 50 + 50/2 + 50/4= 87cm; a los 4 aos: 2 x talla al nacimiento.

101. La forma ms frecuente de hiperplasia suprarrenal congnita es la deficiencia de: (A) 17-hidroxilasa (B) 21-hidroxilasa (C) 11-hidroxilasa (D) 3-Beta-hidroxiesteroide

Nelson, Tratado de Pediatra 15 Ed.,vol. 2 pgs. 2011-2013. La HSC refiere a las deficiencias enzimticas de las vas de biosntesis de los esteroides suprarrenales, que provocan un dficit del glucocorticoide cortisol, acompaado o no del de aldosterona. Dicho dficit determina un aumento de la secrecin de corticotropina, que a su vez ocasiona hiperplasia de la corteza suprarrenal y una produccin excesiva de metabolitos intermedios. El dficit de 21-hidroxilasa es la causante en el 95% de todos los pacientes afectados.

102. El cncer broncgeno ms frecuente en Mxico es el: (A) Epidermoide. (B) Adenocarcinoma. (C) Clulas pequeas. (D) Clulas grandes.

Bibliografa: Manual CTO 5 edicin, Neumologa y Ciruga Torcica, pg 44. Harrison XVI edicin, pg 567. El cncer broncognico ms frecuente en Mxico fue el epidermoide hasta 1975 aproximadamente, pero en los ltimos aos hemos visto aumentar la frecuencia del adenocarcinoma siendo actualmente el tipo histolgico predominante dentro de las neoplasias pulmonares. La mxima incidencia del cncer pulmonar es alrededor de los 60 aos. Recuerda que el cncer de pulmn est claramente relacionado con el consumo de tabaco, en particular el epidermoide. En los no fumadores el ms frecuente es el adenocarcinoma. En cuanto a la localizacin recuerda que el epidermoide suele ser central y tiene tendencia a cavitarse

46

mientras que el adenocarcinoma suele ser perifrico, sin cavitacin, y a veces con afectacin pleural. No pierdas de vista el resto de tipos histolgicos de cncer pulmonar: el carcinoma anaplsico de clulas pequeas(oat cell) y el carcinoma anaplsico de clulas grandes (el carcinoma bronquioloalveolar se considera actualmente un subtipo histolgico del adenocarcinoma

103. En pancreatitis los criterios de Ranson al ingreso incluyen diversos rangos, excepto: (A) Dficit de base menor de 4 mEq/L (B) Edad mayor de 55 aos (C) Leucocitosis mayor de 16,000/mm3 (D) Glucosa mayor de 200 mg/dl

Bruce E. Jarrell, R. Anthony Carabasi, Nacional Medical Series for Independent Study. Wiliams & Wilkins, 3rd Edition: 231-264. Los criterior de Ranson al ingreso incluyen, edad mayor a 55 aos, leucocitosis mayor a 16,000/mm3, Glucosa mayor a 200mg/dl, DHLmayor a 350 UI/L, TGO mayor a 250 SF unidades %. Los criterios a las 48 horas incluyen Descenso del hematocrito mayor a 10 puntos porcentuales, BUN mayor a 5 mg/dl, Ca menor de 8 g/dl, Dficit de base mayor a 4 mEq/L

104. En la esquizofrenia, los trastornos sensoperceptivos ms frecuentes son las alucinaciones: (A) Tactiles. (B) Visuales. (C) Auditivas. (D) Olfatorias.

Manual CTO 5 edicin, Psiquiatra, pg 25 Principios de Medicina Interna Harrison 15 Ed. Pag 2988. Las alucinaciones son trastornos de la percepcin. Recuerda que aparecen sin objeto (o estmulo). En la esquizofrenia los ms frecuentes son los auditivos, mandatorios o que comentan cosas sobre el paciente. Tambin pueden aparecer los tctiles (tienen bichos) o sensaciones en la piel. Otros pueden ser somticos que les hace creer que alguna parte del cuerpo cambia. Los olfatorios son raros, pero tambin pueden aparecer, siendo ms tpicos de las crisis parciales complejas. Es importante que asocies alucinaciones visuales a enfermedades orgnicas, por lo que en esos casos debes descartar patologas endocrinolgicas, neurolgicas, txicas...Recuerda que en la esquizofrenia aparecen trastornos de la forma (ecolalia, perseveracin..) y contenido del pensamiento (ideas delirantes)

47

105. La causa ms frecuente de demencia en Mxico es la: (A) enfermedad multiinfarto. (B) enfermedad de Alzheimer. (C) vascular. (D) enfermedad de parkinson.

Manual CTO 5 edicin, Geriatra, pg 26 . Harrison XVI edicin, pg 2633 La demencia es un sndrome adquirido, producido por una patologa orgnica que consiste en un deterioro persistente de las funciones mentales superiores que conlleva una incapacidad funcional (social y laboral) y cursa sin alteraciones del nivel de conciencia. La prevalencia de las demencias aumenta exponencialmente con la edad. Cuatro tipos de demencia (la enfermedad de Alzheimer, la demencia vascular, la demencia por cuerpos de Lewy difusos y la demencia frontotemporal) causan el 90% de los casos, siendo la enfermedad de Alzheimer la forma ms frecuente de demencia (50% de stas).

106. Cul de las siguientes aseveraciones es cierta para la tiroiditis de Hashimoto? (A) Se asocia a exposicin a radiacin (B) Incrementa el riesgo de cncer (C) Los corticoesteroides son tiles para controlar la progresin de la enfermedad. (D) Es una enfermedad autoinmune.

McPhee S, Papadakis M, Tierney L. Currrent Medical diagnosis and treatment. McGraw Hill 2007 1167-1169. La tiroiditis de Hashimoto tambin conocida como tiroiditis autoinmune o linfoctica crnica. Progresa a hipotiroidismo, lo cual puede deberse a la presencia de anticuerpos contra el receptor de tirotropina. La elevacin srica de anticuerpos contra la peroxidasa tiroidea predicen la progresin de un hipotiroidismo subclnico a un hipotiroidismo sintomtico. Los pacientes presentan niveles elevados de anticuerpos antiperoxidasa en el 90% de los casos o antitiroglobulina en el 40% se puede asociar con enfermedad de addison, hipoparatiroidismo, diabetes, anemia perniciosa, cirrosis biliar, vitiligo y otras condiciones autoinmunes.

48

107. Femenino de 52 aos de edad, con diabetes tipo 2. Se ha controlado bien y ha mantenido su hemoglobina glucosilada en 7% (normal 4-7%) A cul de las siguientes complicaciones sigue ella estando en riesgo a pesar del excelente control de glucosa? (A) Disfuncin autonmica (B) Enfermedad coronaria (C) Ceguera (D) Neuropata perifrica

McPhee S, Papadakis M, Tierney L. Currrent Medical diagnosis and treatment. McGraw Hill 2007 1250. La cardiopata en los pacientes diabticos se debe principalmente a ateroesclerosis coronaria. El riesgo se debe a una combinacin de los siguientes factores: hiperglucemia, Hiperlipidemia, anormalidades en al adhesin plaquetaria, factores de coagulacin, hipertensin, estrs oxidativo e inflamacin.

108. Cul de las siguientes alteraciones se ven en la enfermedad de Addison? (A) Hipernatremia (B) Hiperkalemia (C) BUN bajo (D) Orina diluda

McPhee S, Papadakis M, Tierney L. currrent Medical diagnosis and treatment. McGraw Hill 2007 1189. 1190. La enfermedad de Addison se produce por disfuncin o destruccin de las glndulas suprarrenales, se caracteriza por deficiencia de cortisol, aldosterona y andrgenos. Se observa hiponatremia, deplecin de volumen e hiperkalemia. Puede presentarse Hipercalcemia e hipoglucemia, as como moderada neutropenia, linfocitosis y una cuenta de eosinfilos de 300/mcl. Los sntomas incluyen debilidad, fatiga, prdida de peso, mialgias, artralgias, fiebre,

49

anorexia, nusea, vmito, hipotensin, ansiedad e irritabilidad mental. As hiperpigmentacin de cuello, codos, rodillas, palmas, pezones y las lneas de presin.

como

109. Paciente femenino de 23 aos, que va a ser intervenida quirrgicamente con antecedente de hepatitis. Cul de los siguientes anestsicos est contraindicado en su caso? (A) Halotano (B) Lidocana (C) Fentanilo (D) Ketamina

Hardman J, Limbird L. Goodman and Gilman Las bases farmacolgicas de la teraputica. Mc Graw Hill 2002. 361. El halotano reduce el flujo sanguneo esplcnico y heptico como consecuencia de una reduccin de la presin de riego. Puede producir necrosis heptica fulminate, que se caracteriza por fiebre, anorexia, nusea y vmito que pueden durar varios das despus de la anestesia y acompaarse por un exantema y eosinofilia perifrica. Este sndrome recibe el nombre de hepatitis por halotano.

110. Cul de los siguientes frmacos tiene efecto antabus? (A) Clindamicina (B) Digital (C) Metronidazol (D) Cefotetn Hardman J, Limbird L. Goodman and Gilman Las bases farmacolgicas de la teraputica. Mc Graw Hill 2002.1124. El metronidazol presenta, al igual que el disulfiram, efecto antabus que consiste en molestias abdominales, vmitos, rubor o cefalea si se consumen bebidas alcohlicas durante el tratamiento con este frmaco o tres das despus del mismo.

50

51

S-ar putea să vă placă și